Что делать сначала умножение или сложение: Порядок действий в Математике

Порядок действий в Математике

Статья находится на проверке у методистов Skysmart.
Если вы заметили ошибку, сообщите об этом в онлайн-чат
(в правом нижнем углу экрана).

Основные операции в математике

Основные операции, которые используют в математике — это сложение, вычитание, умножение и деление. Помимо этих операций есть ещё операции отношения, такие как равно (=), больше (>), меньше (<), больше или равно (≥), меньше или равно (≤), не равно (≠).

Операции действия:

  • сложение (+)
  • вычитание (-)
  • умножение (*)
  • деление (:)

Операции отношения:

  • равно (=)
  • больше (>)
  • меньше (<)
  • больше или равно (≥)
  • меньше или равно (≤)
  • не равно (≠)

Сложение — операция, которая позволяет объединить два слагаемых.

  • Запись сложения: 5 + 1 = 6, где 5 и 1 — слагаемые, 6 — сумма.

Вычитание — действие, обратное сложению.

  • Запись вычитания: 10 — 1 = 9, где 10 — уменьшаемое, 1 — вычитаемое, 9 — разность.

Если разность 9, сложить с вычитаемым 1, то получится уменьшаемое 10. Операция сложения 9 + 1 = 10 является контрольной проверкой вычитания 10 — 1 = 9.


Умножение — арифметическое действие в виде краткой записи суммы одинаковых слагаемых.

  • Запись: 3 * 4 = 12, где 3 — множимое, 4 — множитель, 12 — произведение.
  • 3 * 4 = 3 + 3 + 3 + 3

В случае, если множимое и множитель поменять ролями, произведение остается одним и тем же. Например: 5 * 2 = 5 + 5 = 10.

Поэтому и множитель, и множимое называют сомножителями.

Деление — арифметическое действие обратное умножению.

  • Запись: 30 : 6 = 5 или 30/6 = 5, где 30 — делимое, 6 — делитель, 5 — частное.

В этом случае произведение делителя 6 и частного 5, в качестве проверки, дает делимое 30.

4 = 81 — возведение числа 3 в четвертую степень дает 81 (проверка извлечения корня).
  • 2√16 = 4 — корень второй степени называется — квадратным.
  • При знаке квадратного корня показатель корня принято опускать: √16 = 4.

    3√8 = 2 — корень третьей степени называется — кубическим.

    Сложение и вычитание, умножение и деление, возведение в степень и извлечение корня попарно представляют обратные друг другу действия. Далее узнаем порядок выполнения арифметических действий.

     

    Порядок вычисления простых выражений

    Есть однозначное правило, которое определяет порядок выполнения действий в выражениях без скобок:

    • действия выполняются по порядку слева направо
    • сначала выполняется умножение и деление, а затем — сложение и вычитание.

    Из этого правила становится яснее, какое действие выполняется первым. Универсального ответа нет, нужно анализировать каждый пример и подбирать ход решения самостоятельно.

    Что первое, умножение или деление? — По порядку слева направо.

    Сначала умножение или сложение? — Умножаем, потом складываем.

    Порядок выполнения действий в математике (слева направо) можно объяснить тем, что в нашей культуре принято вести записи слева направо. А необходимость сначала умножить или разделить объясняется самой сутью этих операций.

    Рассмотрим порядок арифметических действий в примерах.

    Пример 1. Выполнить вычисление: 11- 2 + 5.

    Как решаем:

    В нашем выражении нет скобок, умножение и деление отсутствуют, поэтому выполняем все действия в указанном порядке. Сначала вычтем два из одиннадцати, затем прибавим к остатку пять и в итоге получим четырнадцать.

    Вот запись всего решения: 11- 2 + 5 = 9 + 5 = 14.

    Ответ: 14.

    Пример 2. В каком порядке выполнить вычисления в выражении: 10 : 2 * 7 : 5?

    Как рассуждаем:

    Чтобы не ошибиться, перечитаем правило для выражений без скобок. У нас есть только умножение и деление — значит сохраняем записанный порядок вычислений и считаем последовательно слева направо.

    Сначала выполняем деление десяти на два, результат умножаем на семь и получившееся в число делим на пять.

    Запись всего решения выглядит так: 10 : 2 * 7 : 5 = 5 * 7 : 5 = 35 : 5 = 7.

    Ответ: 7.

    Пока новые знания не стали привычными, чтобы не перепутать последовательность действий при вычислении значения выражения, удобно над знаками арифметический действий расставить цифры, которые соответствуют порядку их выполнения.

    Например, в такой последовательности можно решить пример по действиям:


    Действия первой и второй ступени

    В некоторых учебниках по математике можно встретить разделение арифметических действий на действия первой и второй ступени.

    • Действиями первой ступени называют сложение и вычитание, а умножение и деление — действиями второй ступени.

    С этими терминами правило определения порядка выполнения действий звучит так:

    Если выражение не содержит скобок, то по порядку слева направо сначала выполняются действия второй ступени (умножение и деление), затем — действия первой ступени (сложение и вычитание).


    Порядок вычислений в выражениях со скобками

    Иногда выражения могут содержать скобки, которые подсказывают порядок выполнения математических действий. В этом случае правило звучит так:

    Сначала выполнить действия в скобках, при этом также по порядку слева направо выполняется умножение и деление, затем — сложение и вычитание.

    Выражения в скобках рассматриваются как составные части исходного выражения. В них сохраняется уже известный нам порядок выполнения действий.

    Рассмотрим порядок выполнения действий на примерах со скобками.

    Пример 1. Вычислить: 10 + (8 — 2 * 3) * (12 — 4) : 2.

    Как правильно решить пример:

    Выражение содержит скобки, поэтому сначала выполним действия в выражениях, которые заключены в эти скобки.

    Начнем с первого 8 — 2 * 3. Что сначала, умножение или вычитание? Мы уже знаем правильный ответ: умножение, затем вычитание. Получается так:

    8 — 2 * 3 = 8 — 6 = 2.

    Переходим ко второму выражению в скобках 12 — 4. Здесь только одно действие – вычитание, выполняем: 12 — 4 = 8.

    Подставляем полученные значения в исходное выражение:

    10 + (8 — 2 * 3) * (12 — 4) : 2 = 10 + 2 * 8 : 2.

    Порядок действий: умножение, деление, и только потом — сложение. Получится:

    10 + 2 * 8 : 2 = 10 + 16 : 2 = 10 + 8 = 18.

    На этом все действия выполнены.

    Ответ: 10 + (8 — 2 * 3) * (12 — 4) : 2 = 18.

    Можно встретить выражения, которые содержат скобки в скобках. Для их решения, нужно последовательно применять правило выполнения действий в выражениях со скобками. Удобнее всего начинать выполнение действий с внутренних скобок и продвигаться к внешним. Покажем на примере.

    Пример 2. Выполнить действия в выражении: 9 + (5 + 1 + 4 * (2 + 3)).

    Как решаем:

    Перед нами выражение со скобками. Это значит, что выполнение действий нужно начать с выражения в скобках, то есть, с 5 + 1 + 4 * (2 + 3). Но! Это выражение также содержит скобки, поэтому начнем сначала с действий в них:

    2 + 3 = 5.

    Подставим найденное значение: 5 + 1 + 4 * 5. В этом выражении сначала выполняем умножение, затем — сложение:

    5 + 1 + 4 * 5 = 5 + 1 + 20 = 26.

    Исходное значение, после подстановки примет вид 9 + 26, и остается лишь выполнить сложение: 9 + 26 = 35.

    Ответ: 9 + (5 + 1 + 4 * (2 + 3)) = 35.

     

    Порядок вычисления в выражениях со степенями, корнями, логарифмами и иными функциями

    Если в выражение входят степени, корни, логарифмы, синус, косинус, тангенс и котангенс, а также другие функции — их значения нужно вычислить до выполнения остальных действий. При этом важно учитывать правила из предыдущих пунктов, которые задают очередность действий в математике.

    Другими словами, перечисленные функции по степени важности можно приравнивать к выражению в скобках.

    И, как всегда, рассмотрим, как это работает на примере.

    Пример 1. Вычислить (4 + 1) * 3 + 62 : 3 — 7.

    Как решаем:

    В этом выражении есть степень 62. И нам нужно найти ее значение до выполнения остальных действий. Выполним возведение в степень: 62 = 36.

    Подставляем полученное значение в исходное выражение:

    (4 + 1) * 3 + 36 : 3 — 7.

    Дальше нам уже все знакомо: выполняем действия в скобках, далее по порядку слева направо выполняем сначала умножение, деление, а затем — сложение и вычитание. Ход решения выглядит так:

    (4 + 1) * 3 + 36 : 3 — 7 = 3 * 3 + 36 : 3 — 7 = 9 + 12 — 7 = 14.

    Ответ: (3 + 1) * 2 + 62 : 3 — 7 = 14.

    Закрепить на практике тему «Порядок действий» можно на курсах по математике в Skysmart!

    Урок 10. порядок выполнения действий в числовых выражениях — Математика — 3 класс

    Математика, 3 класс

    Урок №10. Порядок выполнения действий в числовых выражениях

    Перечень вопросов, рассматриваемых в теме:

    — В какой последовательности выполняются действия в выражениях без скобок?

    — В какой последовательности выполняются действия в выражениях со скобками?

    Глоссарий по теме:

    Если в выражение без скобок входят только сложение и вычитание или умножение и деление, то действия выполняются по порядку: слева направо.

    Если в выражение без скобок входят не только сложение и вычитание, но и умножение или деление, то сначала выполняются по порядку умножение и деление, а затем сложение и вычитание также по порядку.

    Если в выражение есть скобки, то сначала выполняются действия в скобках, а затем в установленном порядке сначала умножение и деление, затем сложение и вычитание

    Основная и дополнительная литература по теме урока:

    1. Моро М. И., Бантова М. А. и др. Математика 3 класс. Учебник для общеобразовательных организаций М.; Просвещение, 2017. – с. 24.

    2. Моро М. И., Волкова С. И. Математика. Рабочая тетрадь 3 класс. Часть 1. М.; Просвещение, 2016. – с. 15.

    Теоретический материал для самостоятельного изучения

    Выполним вычисления устно и расставим значения выражений в порядке возрастания.

    Подсказка: Он должен быть в доме, в шкафу, на столе и даже в портфеле ученика.

    В результате вычислений получилось:

    Действительно во всём должен быть порядок и в математике тоже.

    Выполняя задания, мы пользуемся законами и правилами математики. Эти правила и законы и поддерживают математический порядок.

    Выполняя устные вычисления, мы выполняли действия по порядку. В выражениях использовали действия умножения и деления.

    Рассмотрим выражения:

    6 ∙ 3 + 4 : 2; 27 : 3 — 2 ∙ 2; 2 ∙ (5 + 4).

    Это числовые выражения. Для их составления использовали числа и знаки действий.

    Использовали не только умножение и деление, но и сложение, вычитание. В каком порядке будем выполнять действия?

    В выражении 76 – 27 + 9 – 10 использовали знаки сложения и вычитания. Выполнять действия нужно по порядку: слева направо.

    В выражении 80 : 8 ∙ 2 использовали знаки умножения и деления. Выполнять действия нужно также по порядку: слева направо.

    Вывод: Если в выражениях только сложение и вычитание или умножение и деление, то действия выполняются по порядку слева направо.

    Выражения могут содержать сложение и вычитание, и умножение, и деление. В этом случае сначала выполняются деление и умножение по порядку. В математике эти действия считаются сильными. А затем сложение и вычитание тоже по порядку.

    В математике есть способ, который позволяет выделить какое-то действие. Это постановка скобок. Скобки показывают, что действие внутри них, выполняется в первую очередь.

    Действия в числовых выражениях выполняются в следующем порядке:

    1. Действия записанные в скобках;
    2. Умножение иделение по порядку: слева направо;
    3. Сложение и вычитание по порядку: слева направо.

    Знания этих математических правил позволит правильно находить значения выражений и не нарушать порядок.

    Порядок действий в выражениях особый. 
    И в каждом случае, помните, он свой. 
    В порядке все действия выполняйте.

    Сначала в скобках все посчитайте.

    Потом чередом, умножайте или делите.

    И, наконец, вычитайте или сложите.

    Тренировочные задания.

    1. Выберите действие, которое будет в выражение первым.

    38 + 4 ∙ 7 + 19

    Правильный ответ: умножение.

    2. Выберите действие, которое в выражение будет последним.

    40 : 5 + 12 – 8 : 2

    Правильный ответ: вычитание.

    Порядок выполнения математических действий | интернет проект BeginnerSchool.ru

    Сегодня мы поговорим о порядке выполнения математических действий. Какие действия выполнять первыми? Сложение и вычитание, или умножение и деление. Странно, но у наших детей возникают проблемы с решением, казалось бы, элементарных выражений.

    Читаем выражение слева направо и выбираем порядок действий по приоритету. Сначала выполняем действия в скобках. Затем умножение и/или деление. Далее складываем и вычитаем.

    Если скобки имеют несколько вложений, то есть если внутри скобок есть ещё скобки, то сначала выполняем действия во внутренних скобках. Для простоты понимания, выражение в скобках можно воспринимать как самостоятельное выражение, то есть как отдельный пример, который надо решить. Внутри скобок действия выполняются согласно тому же порядку: Действия в скобках, затем умножение/деление, затем сложение/вычитание.

    Умножение и деление не имеет между собой приоритета и выполняются слева направо, также как и сложение с вычитанием.

    Рассмотрим пример:

    38 – (10 + 6) = 22;

    Итак, вспомним о том, что сначала вычисляются выражения в скобках

    1) в скобках: 10 + 6 = 16;

    2) вычитание: 38 – 16 = 22.

    Если в выражение без скобок входит только сложение и вычитание, или только умножение и деление, то действия выполняются по порядку слева направо.

    10 ÷ 2 × 4 = 20;

    Порядок выполнения действий:

    1) слева направо, сначала деление: 10 ÷ 2 = 5;

    2) умножение: 5 × 4 = 20;

    10 + 4 – 3 = 11, т.е.:

    1) 10 + 4 = 14;

    2) 14 – 3 = 11.

    Если в выражении без скобок есть не только сложение и вычитание, но и умножение или деление, то действия выполняются по порядку слева направо, но преимущество имеет умножение и деление, их выполняют в первую очередь, а за ними и сложение с вычитанием.

    18 ÷ 2 – 2 × 3 + 12 ÷ 3 = 7

    Порядок выполнения действий:

    1) 18 ÷ 2 = 9;

    2) 2 × 3 = 6;

    3) 12 ÷ 3 = 4;

    4) 9 – 6 = 3; т.е. слева направо – результат первого действия минус результат второго;

    5) 3 + 4 = 7; т.е. результат четвертого действия плюс результат третьего;

    Если в выражении есть скобки, то сначала выполняются выражения в скобках, затем умножение и деление, а уж потом сложение с вычитанием.

    30 + 6 × (13 – 9) = 54, т.е.:

    1) выражение в скобках: 13 – 9 = 4;

    2) умножение: 6 × 4 = 24;

    3) сложение: 30 + 24 = 54;

    Итак, подведем итоги. Прежде чем приступить к вычислению, надо проанализировать выражение: есть ли в нем скобки и какие действия в нем имеются. После этого приступать к вычислениям в следующем порядке:

    1)      действия, заключенные в скобках;

    2)      умножение и деление;

    3)      сложение и вычитание.

    Если вы хотите получать анонсы наших статей подпишитесь на рассылку “Новости сайта“.

    Понравилась статья — поделитесь с друзьями:

    Оставляйте пожалуйста комментарии в форме ниже

    Порядок выполнения действий в выражениях без скобок и со скобками — РОСТОВСКИЙ ЦЕНТР ПОМОЩИ ДЕТЯМ № 7

    Сначала сложение или умножение

    С самого начала следует напомнить, чтобы потом не путаться: есть цифры – их 10. От 0 до 9. Есть числа, и они состоят их цифр. Чисел бесконечно много. Точно больше, чем звезд на небе.

    Математическое выражение − это записанное с помощью математических символов наставление, какие действия нужно произвести с числами, чтобы получить результат. Не «выйти» на искомый результат, как в статистике, а узнать, сколько их точно было. А вот чего и когда было − уже не входит в сферу интересов арифметики. При этом важно не ошибиться в последовательности действий, что сначала — сложение или умножение? Выражение в школе иногда называют «пример».

    Сложение и вычитание

    Какие же действия можно произвести с числами? Есть два базовых. Это сложение и вычитание. Все остальные действия построены на этих двух.

    Самое простое человеческое действие: взять две кучки камней и смешать их в одну. Это и есть сложение. Для того чтобы получить результат такого действия, можно даже не знать, что такое сложение.

    Достаточно просто взять кучку камней у Пети и кучку камней у Васи. Сложить все вместе, посчитать все заново. Новый результат последовательного счета камней из новой кучки − это и есть сумма.

    Точно так же можно не знать, что такое вычитание, просто взять и разделить кучу камней на две части или забрать из кучи какое-то количество камней. Вот и останется в куче то, что называется разностью. Забрать можно только то, что есть в куче. Кредит и прочие экономические термины в данной статье не рассматриваются.

    Чтобы не пересчитывать каждый раз камни, ведь бывает, что их много и они тяжелые, придумали математические действия: сложение и вычитание. И для этих действий придумали технику вычислений.

    Сумма двух любых цифр тупо заучиваются без всякой техники. 2 плюс 5 равно семь. Посчитать можно на счетных палочках, камнях, рыбьих головах – результат одинаковый. Положить сначала 2 палочки, потом 5, а потом посчитать все вместе. Другого способа нет.

    Те, кто поумнее, обычно это кассиры и студенты, заучивают больше, не только сумму двух цифр, но и суммы чисел. Но самое главное, они могут складывать числа в уме, используя разные методики. Это называется навыком устного счета.

    Для сложения чисел, состоящих из десятков, сотен, тысяч и еще больших разрядов, используют специальные техники − сложение столбиком или калькулятор. С калькулятором можно не уметь складывать даже цифры, да и читать дальше не нужно.

    Сложение столбиком −­­­­­­ это метод, который позволяет складывать большие (многоразрядные) числа, выучив только результаты сложения цифр. При сложении столбиком последовательно складываются соответствующие десятичные разряды двух чисел (то есть фактически две цифры), если результат сложения двух цифр превышает 10, то учитывается только последний разряд этой суммы – единицы числа, а к сумме следующих разрядов добавляется 1.

    Умножение

    Математики любят группировать похожие действия для упрощения расчетов. Так и операция умножения является группировкой одинаковых действий – сложения одинаковых чисел. Любое произведение N x M − есть N операций сложения чисел M. Это всего лишь форма записи сложения одинаковых слагаемых.

    Для вычисления произведения используется такой же метод – сначала тупо заучивается таблица умножения цифр друг на друга, а потом применяется метод поразрядного умножения, что называется «в столбик».

    Что сначала — умножение или сложение?

    Любое математическое выражение – это фактически запись учетчика «с полей» о результатах каких-либо действий. Допустим, сбора урожая помидоров:

    • 5 взрослых работников собрали по 500 помидоров каждый и выполнили норму.
    • 2 школьников не ходили на уроки математики и помогали взрослым: собрали по 50 помидоров, норму не выполнили, съели 30 помидоров, надкусили и испортили еще 60 помидоров, 70 помидоров было изъято из карманов помощников. Зачем брали с собой их в поле – непонятно.

    Все помидоры сдавали учетчику, он укладывал их по кучкам.

    Запишем результат «сбора» урожая в виде выражения:

    • 500 + 500 + 500 + 500 + 500 — это кучки взрослых работников;
    • 50 + 50 – это кучки малолетних работников;
    • 70 – изъято из карманов школьников (испорченное и надкусанное в зачет результата не идет).

    Получаем пример для школы, запись учетчика результатов работы:

    500 + 500 +500 +500 +500 + 50 +50 + 70 =?;

    Здесь можно применить группировку: 5 кучек по 500 помидоров − это можно записать через операцию умножения: 5 ∙ 500.

    Две кучки по 50 – это тоже можно записать через умножение.

    И одна кучка 70 помидоров.

    5 ∙ 500 + 2 ∙ 50 + 1 ∙ 70 =?

    И что делать в примере сначала − умножение или сложение? Так вот, складывать можно только помидоры. Нельзя сложить 500 помидоров и 2 кучки. Они не складываются. Поэтому сначала нужно всегда все записи привести к базовым операциям сложения, то есть в первую очередь вычислить все операции группировки-умножения. Совсем простыми словами — сначала выполняется умножение, а сложение уже потом. Если умножить 5 кучек по 500 помидоров каждая, то получится 2500 помидоров. А дальше их уже можно складывать с помидорами из других кучек.

    2500 + 100 + 70 = 2 670

    При изучении ребенком математики нужно донести до него, что это инструмент, используемый в повседневной жизни. Математические выражения являются, по сути (в самом простом варианте начальной школы), складскими записями о количестве товаров, денег (очень легко воспринимается школьниками), других предметов.

    Соответственно, любое произведение – это сумма содержимого некоторого количества одинаковых емкостей, ящиков, кучек, содержащих одинаковое количество предметов. И что сначала умножение, а сложение потом, то есть сначала начала вычислить общее количество предметов, а затем уже складывать их между собой.

    Деление

    Операция деления отдельно не рассматривается, она обратная умножению. Нужно что-то распределить по коробкам, так, чтобы во всех коробках было одинаковое заданное количество предметов. Самый прямой аналог в жизни – это фасовка.

    Скобки

    Большое значение в решении примеров имеют скобки. Скобки в арифметике – математический знак, используемый для регулирования последовательности вычислений в выражении (примере).

    Умножение и деление имеют приоритет выше, чем сложение и вычитание.

    А скобки имеют приоритет выше, чем умножение и деление.

    Все, что записано в скобках, вычисляется в первую очередь. Если скобки вложенные, то сначала вычисляется выражение во внутренних скобках. И это непреложное правило. Как только выражение в скобках вычислено, скобки пропадают, а на их месте возникает число. Варианты раскрытия скобок с неизвестными здесь не рассматриваются. Так делают до тех пор, пока все они не исчезнут из выражения.

    1. Это как коробочки с конфетами в большом мешке. Сначала нужно раскрыть все коробочки и ссыпать в большой мешок: (25 – 5 ) = 20. Пять конфет из коробочки сразу заслали отличнице Люде, которая приболела и в празднике не участвует. Остальные конфеты − в мешок!
    2. Потом связать конфеты в пучки по 5 штук: 20 : 5 = 4.
    3. Потом добавить в мешок еще 2 пучка конфет, чтобы можно было поделить на троих детей без драки. Признаки деления на 3 в данной статье не рассматриваются.

    (20 : 5 + 2) : 3 = (4 +2) : 3 = 6 : 3 = 2

    Итого: трем детям по два пучка конфет (по пучку в руку), по 5 конфет в пучке.

    Если вычислить первые скобки в выражении и переписать все заново, пример станет короче. Метод не быстрый, с большим расходом бумаги, зато удивительно эффективный. Заодно тренирует внимательность при переписывании. Пример приводится к виду, когда остается только один вопрос, сначала умножение или сложение без скобок. То есть к такому виду, когда скобок уже и нет. Но ответ на этот вопрос уже есть, и нет смысла обсуждать, что идет сначала — умножение или сложение.

    «Вишенка на торте»

    И напоследок. К математическому выражению не применимы правила русского языка – читать и выполнять слева направо:

    Это простенький пример может довести до истерики ребенка или испортить вечер его маме. Потому что именной ей придется объяснять второкласснику, что бывают отрицательные числа. Или рушить авторитет «МарьиВановны», которая сказала, что: «Нужно слева направо и по порядку».

    «Совсем вишня»

    В Сети гуляет пример, вызывающий затруднения у взрослых дяденек и тетенек. Он не совсем по рассматриваемой теме, что сначала — умножение или сложение. Он вроде как про то, что сначала выполняете действие в скобках.

    От перестановки слагаемых сумма не изменяется, от перестановки множителей тоже. Нужно просто записывать выражение так, чтобы не было потом мучительно стыдно.

    6 : 2 ∙ (1+2) = 6 ∙ ½ ∙ (1+2) = 6 ∙ ½ ∙ 3 = 3 ∙ 3 = 9

    Когда мы работаем с различными выражениями, включающими в себя цифры, буквы и переменные, нам приходится выполнять большое количество арифметических действий. Когда мы делаем преобразование или вычисляем значение, очень важно соблюдать правильную очередность этих действий. Иначе говоря, арифметические действия имеют свой особый порядок выполнения.

    В этой статье мы расскажем, какие действия надо делать в первую очередь, а какие после. Для начала разберем несколько простых выражений, в которых есть только переменные или числовые значения, а также знаки деления, умножения, вычитания и сложения. Потом возьмем примеры со скобками и рассмотрим, в каком порядке следует вычислять их. В третьей части мы приведем нужный порядок преобразований и вычислений в тех примерах, которые включают в себя знаки корней, степеней и других функций.

    Порядок вычисления простых выражений

    В случае выражений без скобок порядок действий определяется однозначно:

    1. Все действия выполняются слева направо.
    2. В первую очередь мы выполняем деление и умножение, во вторую – вычитание и сложение.

    Смысл этих правил легко уяснить. Традиционный порядок записи слева направо определяет основную последовательность вычислений, а необходимость сначала умножить или разделить объясняется самой сутью этих операций.

    Возьмем для наглядности несколько задач. Мы использовали только самые простые числовые выражения, чтобы все вычисления можно было провести в уме. Так можно быстрее запомнить нужный порядок и быстро проверить результаты.

    Условие: вычислите, сколько будет 7 − 3 + 6 .

    Решение

    В нашем выражении скобок нет, умножение и деление также отсутствуют, поэтому выполняем все действия в указанном порядке. Сначала вычитаем три из семи, затем прибавляем к остатку шесть и в итоге получаем десять. Вот запись всего решения:

    7 − 3 + 6 = 4 + 6 = 10

    Ответ: 7 − 3 + 6 = 10 .

    Условие: в каком порядке нужно выполнять вычисления в выражении 6 : 2 · 8 : 3 ?

    Решение

    Чтобы дать ответ на этот вопрос, перечитаем правило для выражений без скобок, сформулированное нами до этого. У нас здесь есть только умножение и деление, значит, мы сохраняем записанный порядок вычислений и считаем последовательно слева направо.

    Ответ: сначала выполняем деление шести на два, результат умножаем на восемь и получившееся в итоге число делим на три.

    Условие: подсчитайте, сколько будет 17 − 5 · 6 : 3 − 2 + 4 : 2 .

    Решение

    Сначала определим верный порядок действий, поскольку у нас здесь есть все основные виды арифметических операций – сложение, вычитание, умножение, деление. Первым делом нам надо разделить и умножить. Эти действия не имеют приоритета друг перед другом, поэтому выполняем их в написанном порядке справа налево. То есть 5 надо умножить на 6 и получить 30 , потом 30 разделить на 3 и получить 10 . После этого делим 4 на 2 , это 2 . Подставим найденные значения в исходное выражение:

    17 − 5 · 6 : 3 − 2 + 4 : 2 = 17 − 10 − 2 + 2

    Здесь уже нет ни деления, ни умножения, поэтому делаем оставшиеся вычисления по порядку и получаем ответ:

    17 − 10 − 2 + 2 = 7 − 2 + 2 = 5 + 2 = 7

    Ответ: 17 − 5 · 6 : 3 − 2 + 4 : 2 = 7 .

    Пока порядок выполнения действий не заучен твердо, можно ставить над знаками арифметических действий цифры, означающие порядок вычисления. Например, для задачи выше мы могли бы записать так:

    .

    Если у нас есть буквенные выражения, то с ними мы поступаем точно так же: сначала умножаем и делим, затем складываем и вычитаем.

    Что такое действия первой и второй ступени

    Иногда в справочниках все арифметические действия делят на действия первой и второй ступени. Сформулируем нужное определение.

    К действиям первой ступени относятся вычитание и сложение, второй – умножение и деление.

    Зная эти названия, мы можем записать данное ранее правило относительно порядка действий так:

    В выражении, в котором нет скобок, сначала надо выполнить действия второй ступени в направлении слева направо, затем действия первой ступени (в том же направлении).

    Порядок вычислений в выражениях со скобками

    Скобки сами по себе являются знаком, который сообщает нам нужный порядок выполнения действий. В таком случае нужное правило можно записать так:

    Если в выражении есть скобки, то первым делом выполняется действие в них, после чего мы умножаем и делим, а затем складываем и вычитаем по направлению слева направо.

    Что касается самого выражения в скобках, его можно рассматривать в качестве составной части основного выражения. При подсчете значения выражения в скобках мы сохраняем все тот же известный нам порядок действий. Проиллюстрируем нашу мысль примером.

    Условие: вычислите, сколько будет 5 + ( 7 − 2 · 3 ) · ( 6 − 4 ) : 2 .

    Решение

    В данном выражении есть скобки, поэтому начнем с них. Первым делом вычислим, сколько будет 7 − 2 · 3 . Здесь нам надо умножить 2 на 3 и вычесть результат из 7 :

    7 − 2 · 3 = 7 − 6 = 1

    Считаем результат во вторых скобках. Там у нас всего одно действие: 6 − 4 = 2 .

    Теперь нам нужно подставить получившиеся значения в первоначальное выражение:

    5 + ( 7 − 2 · 3 ) · ( 6 − 4 ) : 2 = 5 + 1 · 2 : 2

    Начнем с умножения и деления, потом выполним вычитание и получим:

    5 + 1 · 2 : 2 = 5 + 2 : 2 = 5 + 1 = 6

    На этом вычисления можно закончить.

    Ответ: 5 + ( 7 − 2 · 3 ) · ( 6 − 4 ) : 2 = 6 .

    Не пугайтесь, если в условии у нас содержится выражение, в котором одни скобки заключают в себе другие. Нам надо только применять правило выше последовательно по отношению ко всем выражениям в скобках. Возьмем такую задачу.

    Условие: вычислите, сколько будет 4 + ( 3 + 1 + 4 · ( 2 + 3 ) ) .

    Решение

    У нас есть скобки в скобках. Начинаем с 3 + 1 + 4 · ( 2 + 3 ) , а именно с 2 + 3 . Это будет 5 . Значение надо будет подставить в выражение и подсчитать, что 3 + 1 + 4 · 5 . Мы помним, что сначала надо умножить, а потом сложить: 3 + 1 + 4 · 5 = 3 + 1 + 20 = 24 . Подставив найденные значения в исходное выражение, вычислим ответ: 4 + 24 = 28 .

    Ответ: 4 + ( 3 + 1 + 4 · ( 2 + 3 ) ) = 28 .

    Иначе говоря, при вычислении значения выражения, включающего скобки в скобках, мы начинаем с внутренних скобок и продвигаемся к внешним.

    Допустим, нам надо найти, сколько будет ( 4 + ( 4 + ( 4 − 6 : 2 ) ) − 1 ) − 1 . Начинаем с выражения во внутренних скобках. Поскольку 4 − 6 : 2 = 4 − 3 = 1 , исходное выражение можно записать как ( 4 + ( 4 + 1 ) − 1 ) − 1 . Снова обращаемся к внутренним скобкам: 4 + 1 = 5 . Мы пришли к выражению ( 4 + 5 − 1 ) − 1 . Считаем 4 + 5 − 1 = 8 и в итоге получаем разность 8 — 1 , результатом которой будет 7 .

    Порядок вычисления в выражениях со степенями, корнями, логарифмами и иными функциями

    Если у нас в условии стоит выражение со степенью, корнем, логарифмом или тригонометрической функцией (синусом, косинусом, тангенсом и котангенсом) или иными функциями, то первым делом мы вычисляем значение функции. После этого мы действуем по правилам, указанным в предыдущих пунктах. Иначе говоря, функции по степени важности приравниваются к выражению, заключенному в скобки.

    Разберем пример такого вычисления.

    Условие: найдите, сколько будет ( 3 + 1 ) · 2 + 6 2 : 3 − 7 .

    Решение

    У нас есть выражение со степенью, значение которого надо найти в первую очередь. Считаем: 6 2 = 36 . Теперь подставим результат в выражение, после чего оно примет вид ( 3 + 1 ) · 2 + 36 : 3 − 7 .

    Дальше действуем по знакомому алгоритму: считаем, сколько у нас получится в скобках, потом в оставшемся выражении выполняем умножение и деление, а следом – сложение и вычитание.

    ( 3 + 1 ) · 2 + 36 : 3 − 7 = 4 · 2 + 36 : 3 − 7 = 8 + 12 − 7 = 13

    Ответ: ( 3 + 1 ) · 2 + 6 2 : 3 − 7 = 13 .

    В отдельной статье, посвященной вычислению значений выражений, мы приводим и другие, более сложные примеры подсчетов в случае выражений с корнями, степенью и др. Рекомендуем вам с ней ознакомиться.

    Сайт для детей и их родителей

    Порядок выполнения математических действий

    Сегодня мы поговорим о порядке выполнения математических действий. Какие действия выполнять первыми? Сложение и вычитание, или умножение и деление. Странно, но у наших детей возникают проблемы с решением, казалось бы, элементарных выражений.

    Итак, вспомним о том, что сначала вычисляются выражения в скобках

    1) в скобках: 10 + 6 = 16 ;

    2) вычитание: 38 – 16 = 22 .

    Если в выражение без скобок входит только сложение и вычитание, или только умножение и деление, то действия выполняются по порядку слева направо.

    Порядок выполнения действий:

    1) слева направо, сначала деление: 10 ÷ 2 = 5 ;

    2) умножение: 5 × 4 = 20 ;

    10 + 4 – 3 = 11 , т.е.:

    Если в выражении без скобок есть не только сложение и вычитание, но и умножение или деление, то действия выполняются по порядку слева направо, но преимущество имеет умножение и деление, их выполняют в первую очередь, а за ними и сложение с вычитанием.

    18 ÷ 2 – 2 × 3 + 12 ÷ 3 = 7

    Порядок выполнения действий:

    4) 9 – 6 = 3 ; т.е. слева направо – результат первого действия минус результат второго;

    5) 3 + 4 = 7 ; т. е. результат четвертого действия плюс результат третьего;

    Если в выражении есть скобки, то сначала выполняются выражения в скобках, затем умножение и деление, а уж потом сложение с вычитанием.

    30 + 6 × (13 – 9) = 54 , т.е.:

    1) выражение в скобках: 13 – 9 = 4 ;

    2) умножение: 6 × 4 = 24 ;

    3) сложение: 30 + 24 = 54 ;

    Итак, подведем итоги. Прежде чем приступить к вычислению, надо проанализировать выражение: есть ли в нем скобки и какие действия в нем имеются. После этого приступать к вычислениям в следующем порядке:

    1) действия, заключенные в скобках;

    2) умножение и деление;

    3) сложение и вычитание.

    Если вы хотите получать анонсы наших статей подпишитесь на рассылку “Новости сайта“.

    1. Математика – 3 классПродолжим изучение предметов, которые изучают наши дети в начальной школе.
    2. Математика – 2 классПродолжим изучение программы математики в начальной школе и на этот.

    Понравилась статья — поделитесь с друзьями:

    Подпишитесь на новости сайта:

    Оставляйте пожалуйста комментарии в форме ниже

    «Порядок действий в выражениях без скобок, имеющих действия разных ступеней»

    (2 класс, Образовательная система «Перспектива»

    Авторы: Дорофеев Г. В., Миракова Т.Н., Бука Т.Б.)

    Тема урока: Порядок действий в выражении без скобок, имеющих действия разных ступеней»

    Цели урока:

    Образовательная: формирование представления о способе выполнения порядка действий в выражениях, имеющих действия разных ступеней. Развивающая: развитие умения наблюдать, сравнивать, анализировать, делать выводы.

    Воспитательная: воспитание взаимопомощи, умения слушать своего товарища, принимать точку зрения другого человека отличную от собственной.

    I Организационный момент.

    — Здравствуйте, ребята! Начинаем урок математики.

    — Если вы знаете о чем идет речь, хлопните в ладоши:

    переместительное свойство умножения;

    таблица умножения и деления на 2, 3, 4;

    компоненты умножения;

    компоненты деления;

    компоненты сложения;

    компоненты вычитания

    — Как много вы уже знаете! А хотите узнать еще больше? Будьте старательными, активными, внимательными и у вас все получится.

    II Актуализация знаний, умений, навыков и способов действий.

    1. – Прочитайте выражения по-разному: (Найти произведение чисел двух и семи; два умножить на семь; первый множитель два, второй множитель — семь, найти произведение; по два взять семь раз и т.д.)

    2х7 15:3 17-3 8+9

    Учитель:— придумайте свои примеры и прочитайте их разными способами. (Работа в паре)

    Дети придумывают свои примеры и читают их разными способами.

    Оценка работы. – Что помогло вам успешно справиться с заданием? (знание компонентов сложения, вычитания, умножения и деления, таблица умножения).

    1. Порядок действий в выражении, где только сложение и вычитание.

    12-2+3=13

    12-2+3=7

    — Чем похожи и чем отличаются записанные равенства? (Одинаковые числа, знаки, различный результат)

    — Назовите истинное высказывание.

    — что нужно сделать, чтобы ложное высказывание стало истинным? (Поставить скобки, в 1 примере скобки можно поставить, можно не ставить, а во 2 ставим обязательно)

    Оценка фронтальной работы. – Что помогло вам успешно справиться с заданием? (знание порядка действий, где только сложение и вычитание без скобок и со скобками).

    3.Предлагаю вам еще решить примеры.

    (На доске записаны примеры.)

    Ваня и Аня решали примеры. Рассмотрите примеры. Сравните. Чем они похожи? Чем отличаются?

    19 – 8 + 5 =16

    2 * 6 : 3 = 4

    2 + 3 * 4 = 20

    19 – 8 + 5 = 16

    2 * 6 : 3 = 4

    2 + 3 * 4 = 14

    — Решите первый пример.

    — Как выполняли действия?  (Сначала выполнили вычитание, а потом сложение.)

    — Решите второй пример.

    — Как выполняли действия. (Сначала выполнили умножение, а потом деление).

    — Как решал третий пример Ваня? (Сначала выполнил сложение, а потом умножение).

    — А как решала Аня? (Сначала выполнила умножение, а потом сложение).

    — Как вы думаете, кто из ребят верно решил третий пример? Обоснуйте свой ответ. (Мы не можем определить, кто решил верно.)

    — Чего мы не знаем?  (Мы не знаем, в каком порядке нужно выполнять действия в некоторых примерах.)

    — Чему вы должны научиться?

    (Решать примеры на порядок действий в выражении без скобок, где есть и сложение, и вычитание, и умножение, и деление)

    — Какая тема урока?  (Порядок выполнения действий.)

    -Какова цель нашего урока? (Научиться устанавливать правильный порядок действий.)

    III Открытие нового способа.

    Откройте учебник на с. 97, прочитайте выделенное в рамочку.

    — Назовите арифметические действия первой ступени. Расположите их на нужной ступени.

    — Назовите арифметические действия второй ступени. Расположите их на нужной ступени.

    — Какая ступень выше? (Вторая)

    — Сделайте вывод: какое действие надо выполнять первым? (умножение и деление) вторым? (сложение и вычитание)

    — Мы получили новое знание. Проверим, правильно ли мы сделали выводы.

    Сравним наше новое знание с научным в учебнике. Прочитайте на с.97 выделенное в рамочку.

    — Как вы думаете, кто верно решил третий пример: Ваня или Аня? Обоснуйте свой ответ. (Аня, т.к. в этом примере сначала надо выполнить действие умножения, а потом сложения)

    IV. Физминутка (из СанПин) зрительная

    V. Первичное закрепление с комментированием

    1. -Попробуем применить наше новое знание на практике.

    (Коллективное выполнение с комментированием.)

    3+5 -2 = 2+2+2=

    2*3+14= 19- 9 – 10=

    4*3:6= 16 – 8 : 2=

    — Выберите примеры с действиями только первой ступени.

    -Как выполнять действия в первом столбике? (По порядку.)

    — Выберите примеры с действиями только второй ступени.

    -Как выполнять действия во втором столбике? (По порядку)

    — Какие примеры остались? (примеры с действиями обеих ступеней)

    — Как будем выполнять действия в третьем столбике? (Сначала выполним действие умножения и деления , а затем сложения и вычитания).

    2. — Давайте проверим, «работает» ли открытый нами способ на других примерах?

    1. Работа по учебнику.

    С. 98, задание№3(1 стр.)

    Взаимопроверка. Молодцы! Что помогло вам успешно справиться с решением примеров?

    VI. Контроль

    1. Обозначьте порядок действий в выражениях:

    а+в ×с-у

    у:с-к+а

    в-а+р:т

    — Сверьте с доской. Самооценка

    — Что вам нужно было сделать?

    — Удалось ли правильно решить?

    — Вы все сделали правильно или были ошибки, недочеты?

    — Вы сделали все сами или с чьей-то помощью?

    — Оцените свою работу?

    Критерии оценивания:

    Все задания выполнены самостоятельно, верно – «5»

    Все задания выполнены, но было обращение за помощью – «4»

    Допущено 1-2 незначительных ошибки – «4»

    Выполнено 1 задание из 3, за помощью не обращался – «3»

    Не выполнил совсем – «2»

    VII Рефлексивная оценка.

    — Вы могли в начале урока правильно решить примеры?

    — Изменились ли ваши знания? Что нового добавилось к уже имеющимся знаниям?

    — Как будете действовать дома при решении примеров, где встречаются разные действия?

    VIII Домашнее задание.

    1. Учебник с.98, пр.3 (2 стр.), №4

    2. Придумать два выражения на порядок действий, используя все четыре действия.

    5

    Зубодробительная задачка с очень простой математикой

    В интер­не­те мно­го спо­ров про такие при­ме­ры, поэто­му мы реши­ли разо­брать­ся, какие ошиб­ки совер­ша­ют чаще все­го и поче­му мно­гие счи­та­ют непра­виль­но. Для реше­ния нам пона­до­бят­ся три мате­ма­ти­че­ских правила:

    1. То, что в скоб­ках, выпол­ня­ет­ся в первую оче­редь. Если ско­бок несколь­ко, они выпол­ня­ют­ся сле­ва направо.
    2. При отсут­ствии ско­бок мате­ма­ти­че­ские дей­ствия выпол­ня­ют­ся сле­ва напра­во, сна­ча­ла умно­же­ние и деле­ние, потом — сло­же­ние и вычитание.
    3. Меж­ду мно­жи­те­лем и скоб­кой (или дву­мя скоб­ка­ми) может опус­кать­ся знак умножения.

    Раз­бе­рём подроб­нее, что это зна­чит в нашем случае.

    1. То, что в скоб­ках, выпол­ня­ет­ся в первую оче­редь. То есть в нашем при­ме­ре, вне зави­си­мо­сти от чего угод­но, сна­ча­ла схлоп­нут­ся скобки:

    8 / 2(2 + 2) → 8 / 2(4)

    2. Меж­ду чис­лом и скоб­кой мож­но опу­стить знак умно­же­ния. У нас перед скоб­кой двой­ка, то есть мож­но сде­лать такую замену:

    8 / 2(4) → 8 / 2 × 4

    3. Мате­ма­ти­че­ские дей­ствия при отсут­ствии ско­бок выпол­ня­ют­ся сле­ва напра­во: как при чте­нии, сна­ча­ла умно­же­ние и деле­ние, потом — сло­же­ние и вычи­та­ние. Умно­же­ние и деле­ние име­ют оди­на­ко­вый при­о­ри­тет. Нет тако­го, что сна­ча­ла все­гда дела­ет­ся умно­же­ние, затем деле­ние, или наобо­рот. Со сло­же­ни­ем и вычи­та­ни­ем то же самое.

    Неко­то­рые счи­та­ют, что раз мно­жи­те­ли были напи­са­ны близ­ко друг к дру­гу (когда там сто­я­ли скоб­ки), то оно выпол­ня­ет­ся в первую оче­редь, ссы­ла­ясь при этом на раз­ные мето­ди­че­ские посо­бия. На самом деле это не так, и нет тако­го скры­то­го умно­же­ния, кото­рое име­ет при­о­ри­тет над дру­гим умно­же­ни­ем или деле­ни­ем. Это такое же умно­же­ние, как и осталь­ные, и оно дела­ет­ся в общем поряд­ке — как и при­ня­то во всём мате­ма­ти­че­ском мире.

    Полу­ча­ет­ся, что нам сна­ча­ла надо сло­жить 2 + 2 в скоб­ках, потом 8 раз­де­лить на 2, и полу­чен­ный резуль­тат умно­жить на то, что в скобках:

    8 / 2 × (2 + 2) = 8 / 2 × 4 = 4 × 4 = 16

    Кста­ти, если на айфоне запи­сать это выра­же­ние точ­но так же, как в усло­вии, теле­фон тоже даст пра­виль­ный ответ.

    А инже­нер­ный каль­ку­ля­тор на Windows 10 так запи­сы­вать не уме­ет и про­пус­ка­ет первую двойку-множитель. Попро­буй­те сами 🙂

    Тут в тред вры­ва­ют­ся мате­ма­ти­ки и с воп­ля­ми «Шустеф!» пояс­ня­ют криком:

    «В АЛГЕБРЕ ТОТ ЖЕ ПОРЯДОК ДЕЙСТВИЙ, ЧТО И В АРИФМЕТИКЕ, но есть исклю­че­ние: в алгеб­ре знак умно­же­ния свя­зы­ва­ет ком­по­нен­ты дей­ствия силь­нее, чем знак деле­ния, поэто­му знак умно­же­ния опус­ка­ет­ся. Напри­мер, a:b·c= a: (b·c)».

    Этот текст из «Мето­ди­ки пре­по­да­ва­ния алгеб­ры», курс лек­ций, Шустеф М. Ф., 1967 год. (стр. 43)

    Раз в спор­ном при­ме­ре знак умно­же­ния опу­щен, то спор­ный при­мер алгеб­ра­и­че­ский, а зна­чит, сна­ча­ла умно­жа­ем 2 на 4, а потом 8 делим на 8!


    Та самая цитата. 

    А вот как на это отве­ча­ют те, кто дей­стви­тель­но в теме и не ленит­ся пол­но­стью посмот­реть первоисточник:

    «Для устра­не­ния недо­ра­зу­ме­ний В. Л. Гон­ча­ров ука­зы­ва­ет, что пред­по­чти­тель­нее поль­зо­вать­ся в каче­стве зна­ка деле­ния чер­той и ста­вить скоб­ки [87]. П. С. Алек­сан­дров и А. Н. Кол­мо­го­ров [59] пред­ло­жи­ли изме­нить поря­док дей­ствий в ариф­ме­ти­ке и решать, напри­мер, так: 80:20×2=80:40=2 вме­сто обыч­но­го: 80:20×2=4×2=8. Одна­ко это пред­ло­же­ние не нашло поддержки».

    Если апел­ли­ро­вать к Фри­де Мак­совне Шустеф, то выхо­дит, что:

    1. В. Л. Гон­ча­ров гово­рит так: «Ребя­та, исполь­зуй­те чер­ту и ставь­те скоб­ки, что­бы ни у кого не было вопро­сов про приоритет».
    2. Если у нас всё же бит­ва ариф­ме­ти­ки и алгеб­ры, то, по П. С. Алек­сан­дро­ву и А. Н. Кол­мо­го­ро­ву, при­мер нуж­но решать сле­ва напра­во, как обыч­но. Они, конеч­но, пред­ло­жи­ли решать такое по-другому, но науч­ное сооб­ще­ство их не поддержало.

    Самое инте­рес­ное, что даль­ше в при­ме­рах Фри­да Мак­сов­на поль­зу­ет­ся как раз пра­виль­ным поряд­ком дей­ствий, объ­яс­няя реше­ние. Даже там, где есть умно­же­ние на скоб­ку с опу­щен­ным зна­ком, она выпол­ня­ет дей­ствия сле­ва направо.


    Пол­ная цита­та из Шустеф, кото­рая, ока­зы­ва­ет­ся, име­ет в виду совсем не то. 

    Приоритет — Python

    Посмотрите внимательно на выражение 2 + 2 * 2 и посчитайте в уме ответ.

    Правильный ответ: 6.

    Если у вас получилось 8, то этот урок для вас. В школьной математике мы изучали понятие «приоритет операции». Приоритет определяет то, в какой последовательности должны выполняться операции. Например, умножение и деление имеют больший приоритет, чем сложение и вычитание, а приоритет возведения в степень выше всех остальных арифметических операций: 2 ** 3 * 2 вычислится в 16.

    Но нередко вычисления должны происходить в порядке, отличном от стандартного приоритета. В сложных ситуациях приоритет можно (и нужно) задавать круглыми скобками, точно так же, как в школе, например: (2 + 2) * 2.

    Скобки можно ставить вокруг любой операции. Они могут вкладываться друг в друга сколько угодно раз. Вот пара примеров:

    print(3 ** (4 - 2))  # => 9
    print(7 * 3 + (4 / 2) - (8 + (2 - 1)))  # => 14
    

    Главное при этом соблюдать парность, то есть закрывать скобки в правильном порядке. Это, кстати, часто становится причиной ошибок не только у новичков, но и у опытных программистов. Для удобства ставьте сразу открывающую и закрывающую скобку, а потом пишите внутреннюю часть. Редактор на нашем сайте (и большинство других редакторов кода) делают это автоматически: вы пишете (, а редактор сразу добавляет ). Это касается и других парных символов, например, кавычек. О них — в будущих уроках.

    Иногда выражение сложно воспринимать визуально. Тогда можно расставить скобки, не повлияв на приоритет. Например, задание из прошлого урока можно сделать немного понятнее, если расставить скобки.

    Было:

    print(8 / 2 + 5 - -3 / 2)  # => 10.5
    

    Стало:

    print(((8 / 2) + 5) - (-3 / 2))  # => 10.5
    

    Запомните: код пишется для людей, потому что код будут читать люди, а машины будут только исполнять его. Для машин код — или корректный, или не корректный, для них нет «более» понятного или «менее» понятного кода.

    Задание

    Дано вычисление 70 * 3 + 4 / 8 + 2.

    Расставьте скобки так, чтобы оба сложения (3 + 4) и (8 + 2) высчитывались в первую очередь. Выведите на экран результат.


    Нашли ошибку? Есть что добавить? Пулреквесты приветствуются https://github.com/hexlet-basics

    Как использовать PEMDAS и решить с порядком операций (примеры)

    Обновлено 21 декабря 2020 г.

    Ли Джонсон

    Выполнение математической задачи, сочетающей в себе различные операции, такие как умножение, сложение и экспоненты, может вызвать недоумение, если вы не сделаете этого. Не понимаю ПЕМДАС. Простая аббревиатура описывает порядок операций в математике, и вы должны помнить ее, если вам нужно выполнять вычисления на регулярной основе. PEMDAS означает круглые скобки, экспоненты, умножение, деление, сложение и вычитание, сообщая вам порядок, в котором вы беретесь за различные части длинного выражения.Узнайте, как это использовать, и вас никогда не смутят проблемы типа 3 + 4 × 5–10, с которыми вы можете столкнуться.

    Совет: PEMDAS описывает порядок операций:

    M и D — умножение и деление

    A и S — сложение и вычитание.

    Решайте любые проблемы с различными типами операций в соответствии с этим правилом, работая сверху (скобки) вниз (сложение и вычитание), отмечая, что операции в одной строке можно выполнять слева направо по мере их появления. в вопросе.

    Каков порядок работы?

    Порядок операций показывает, какие части длинного выражения нужно вычислить в первую очередь, чтобы получить правильный ответ. Если вы, например, просто подходите к вопросам слева направо, в большинстве случаев вы в конечном итоге рассчитываете что-то совершенно другое. PEMDAS описывает порядок операций следующим образом:

    M и D — Умножение и деление

    A и S — Сложение и вычитание.

    Когда вы решаете длинную математическую задачу с помощью множества операций, сначала вычислите что-нибудь в круглых скобках, а затем переходите к показателям степени (т.е., «степени» чисел) перед умножением и делением (они работают в любом порядке, просто работают слева направо). Наконец, вы можете работать над сложением и вычитанием (опять же, просто работайте слева направо).

    Как запомнить PEMDAS

    Запоминание аббревиатуры PEMDAS, вероятно, является самой сложной частью его использования, но есть мнемонические средства, которые вы можете использовать, чтобы упростить это. Самым распространенным является «Прошу прощения, моя дорогая тетя Салли», но есть и другие альтернативы: «Люди повсюду принимают решения о суммах» и «Пухлые эльфы могут потребовать перекус». 3-3) × 4

    Решения перечислены ниже по порядку, поэтому не прокручивайте вниз, пока не попытаетесь решить проблему.3 — 3) × 4 \\ & = 20 ÷ (8 — 3) × 4 \\ & = 20 ÷ 5 × 4 \\ & = 16 \ end {align}

    Правило PEMDAS — Что такое Правило PEMDAS? Определение, примеры

    В математике PEMDAS — это аббревиатура, используемая для обозначения порядка операций, которым необходимо следовать при решении выражений, содержащих несколько операций. PEMDAS означает P- круглые скобки, E- экспоненты, M- умножение, D- деление, A- сложение и S- вычитание. В разных странах используются разные аббревиатуры для обозначения порядка операций.Например, в Канаде порядок операций указан как BEDMAS (скобки, экспоненты, деление, умножение, сложение и вычитание). Некоторые люди предпочитают говорить BODMAS (B- скобки, O- порядок или Off), в то время как немногие другие называют это GEMDAS (G- группировка).

    В этом уроке вы узнаете о правиле PEMDAS для решения арифметических выражений с последующими решенными примерами и практическими вопросами.

    Введение в PEMDAS

    PEMDAS или порядок операций — это установленная последовательность для выполнения операций в арифметическом выражении.Существуют разные сценарии, в которых все проходит через различные этапы в фиксированной последовательности. Рассмотрим следующий сценарий. Райан и Рухи посетили фабрику игрушек. Они оба наблюдали за производством игрушек на фабрике. Сначала конструируют игрушки. Далее их собирают и упаковывают в коробки. Наконец, они проверяются на качество перед отправкой в ​​магазины. Все сделано в установленном порядке.

    Точно так же арифметические операции выполняются упорядоченным образом.Давайте узнаем порядок операций по математике. Найти ответ на математические операции довольно просто, если задействован только один оператор. Что делать, если задействовано несколько операторов? Это могло быть немного сложнее! Посмотрим как.

    Райан и Рухи решили отдельно математическое выражение 5 + 2 × 3. Вот как они это решили.

    Метод Рухи Метод Райана

    5 + 2 × 3

    = 7 × 3

    = 21

    5 + 2 × 3

    = 5 + 6

    = 11

    Как видите, Райан и Рухи получили разные ответы. На это выражение в математике может быть только один правильный ответ! Вы можете решить, кто прав?

    Не волнуйтесь! PEMDAS поможет вам найти правильный ответ.

    Что такое PEMDAS?

    PEMDAS — это порядок операций, используемый в математике для упрощения сложных вычислений. В нем говорится, что мы начинаем решать любое арифметическое выражение, решая члены, записанные в скобках или скобках, а затем упрощаем экспоненциальные члены и переходим к операциям умножения и деления, а затем, наконец, мы можем найти ответ, решая операции сложения и вычитания.

    PEMDAS: определение

    PEMDAS — это набор правил, которым следуют при решении математических выражений. Это правило начинается с скобок , а затем операции выполняются с экспонентами или степенями. Далее выполняем операции умножения или деления слева направо. Наконец, операции сложения или вычитания выполняются слева направо.

    п. [{()}] Круглые скобки
    E х 2 Экспоненты

    м

    D

    ×

    ИЛИ

    ÷

    Умножение

    ИЛИ

    Дивизия

    А

    S

    +

    ИЛИ

    Дополнение

    ИЛИ

    Вычитание

    Если вы будете придерживаться этого порядка операций в правиле PEMDAS, вы всегда получите правильный ответ.Следующая аббревиатура поможет вам запомнить Правило PEMDAS.

    P аренда E xcuse M y D ear A unt S союзник

    Давайте разберемся с PEMDAS на примере.

    BODMAS против PEMDAS

    Правило PEMDAS аналогично правилу BODMAS . Существует различие в сокращении, потому что определенные термины известны под разными именами в разных местах.

    Когда использовать PEMDAS?

    Когда в математическом выражении содержится более одной операции, мы используем метод PEMDAS. PEMDAS в математике дает вам правильную структуру для получения уникального ответа для каждого математического выражения. При использовании метода PEMDAS необходимо соблюдать последовательность определенных правил. Как только вы освоите эти правила, вы сможете выполнять сразу несколько шагов.

    Что следует помнить

    • Операции, указанные в скобках, должны быть выполнены в первую очередь.
    • Затем решите степень в выражении.
    • Двигайтесь слева направо и выполняйте умножение или деление, в зависимости от того, что наступит раньше.
    • Двигайтесь слева направо и выполняйте сложение или вычитание, в зависимости от того, что наступит раньше.

    Распространенные ошибки при использовании правила PEMDAS в математике

    Наличие нескольких скобок обычно вызывает путаницу. Если мы не знаем, какую скобку решить в первую очередь, это может привести к неправильному ответу. Теперь мы узнаем, как решить это выражение с несколькими скобками.

    4 + 3 [8-2 (6-3)] ÷ 2

    Начнем с внутренней стороны скоб. Сначала мы решим самую внутреннюю скобку, а затем двинемся наружу.

    • Начиная с 6 — 3 = 3, получаем: 4 + 3 [8 — 2 (3)] ÷ 2
    • Далее, умножая 2 (3) = 6 или 2 × 3 = 6, получаем: 4 + 3 [8 — 6] ÷ 2
    • Осталась одна скобка, [8 — 6] = 2, получаем: 4 + 3 [2] ÷ 2
    • Решая 3 [2] или 3 × 2 = 6, получаем: 4 + 6 ÷ 2

    Мы видим, что все выражения в скобках решены.Основываясь на PEMDAS, мы знаем, что дальше идет деление, следовательно, 6 ÷ 2 = 3, то есть 4 + 3. И, наконец, сложение 4 + 3 = 7.

    Часто задаваемые вопросы о PEMDAS

    Как работает правило Пемдаса?

    В любом арифметическом выражении, если используется несколько операций, мы должны сначала решить термины, указанные в скобках. Избавившись от скобок, мы решаем операцию умножения и деления, независимо от того, что идет первым в выражении слева направо. Тогда мы получим упрощенное выражение, состоящее только из операций сложения и вычитания. Мы решаем сложение и вычитание слева направо, что бы ни случилось раньше, и получаем окончательный ответ. Так работает PEMDAS.

    Что означает PEMDAS?

    PEMDAS означает порядок операций для математических выражений, включающих более одной операции. Это означает P- круглые скобки, E- экспоненты, M- умножение, D- деление, A- сложение и S- вычитание.

    Как делать Пемдас с дробями?

    В выражении с дробными членами нет изменений в использовании правила PEMDAS.Это нужно делать так же, как и любое целочисленное выражение.

    Что означает буква P в Пемдасе?

    В PEMDAS буква P означает круглые или квадратные скобки. Порядок решения скобок задается как [{()}]. Это означает, что мы всегда сначала решаем самую внутреннюю скобку, а затем переходим к фигурным скобкам и квадратным скобкам.

    Вы сначала умножаете или делите в PEMDAS?

    В правиле PEMDAS мы решаем операции умножения и деления слева направо. Мы можем выполнить любую операцию, умножение или деление, что бы ни было первым в выражении.

    Когда мы применяем правило PEMDAS?

    Правило PEMDAS применяется для решения сложного математического выражения, включающего более одной операции, например сложение, вычитание, умножение или деление.

    Какое правило для PEMDAS?

    Правило PEMDAS дает нам правильную последовательность решения математического выражения. В правиле PEMDAS сначала операции выполняются в скобках.Далее выполняются операции с показателями степени или степенями. Затем следуют операции умножения или деления слева направо, в зависимости от того, что наступит раньше. Наконец, операции сложения или вычитания выполняются слева направо, в зависимости от того, что наступит раньше.

    Что такое BODMAS, BIDMAS и PEMDAS?

    BODMAS объясняет «Порядок операций» в математике, а BIDMAS и PEMDAS делают то же самое, но используют несколько разные слова. Если для вас все вышесказанное звучит как чепуха, взгляните на наше короткое видео ниже, где Али объясняет, как все это работает.


    Что такое аббревиатура?

    Аббревиатура — это слово, состоящее из начальных букв слов во фразе. Аббревиатуры можно произносить как отдельные слова.

    Подумайте о НАСА и FAQ. Оба они произносятся так, как если бы они были самостоятельным словом. Однако NASA означает N ational A eronautics и S pace A dministration and FAQ означает F обязательно A sked Q uestions.

    Подобным образом буквы в BODMAS, BIDMAS и PEMDAS обозначают слова.

    Что обозначают буквы в BODMAS, BIDMAS и PEMDAS?

    В случае математических сокращений BODMAS, BIDMAS и PEMDAS все они означают одно и то же, и все они служат одной и той же цели. Вот что обозначают буквы:

    • Кронштейны
    • Заказы
    • Дивизия
    • Умножение
    • Дополнение
    • Вычитание
    • Кронштейны
    • Индексы
    • Дивизия
    • Умножение
    • Дополнение
    • Вычитание
    • Круглая скобка
    • Показатели
    • Умножение
    • Дивизия
    • Дополнение
    • Вычитание

    Третья серия, приведенная выше, наиболее часто используется в США, в то время как школы Великобритании почти всегда применяют серию 1 или серию 2, указанную выше. Когда ваш ребенок впервые сталкивается с одним из них, вам следует вспомнить, какой именно, потому что в разных школах используются разные. Придерживаясь того же, что и в вашей школе, вы, по крайней мере, избежите одного уровня сложности.

    Что такое ордера, индексы и экспоненты?

    Заказы, индексы и экспоненты — это маленькие числа в верхнем индексе, обозначающие степени в математике.

    Скобки, деление, умножение, сложение и вычитание — это слова, которые нам знакомы, но как насчет второго элемента аббревиатуры — порядков, индексов или экспонентов? Это «числа половинного размера», которые часто встречаются в математических выражениях.В следующем примере 2 — это индекс (единственное число из множественного числа , индексы ):

    3 2 = 3 x 3

    Что такое математические операции?

    Математическая операция может рассматриваться как один из символов , который заставляет что-то случиться в математике. Знак умножения (x) означает, что два числа нужно умножить вместе, а знак сложения (+) означает, что два числа нужно сложить вместе. Это операции .

    Каков правильный порядок действий в математике?

    Правильный порядок операций в математике: B rackets, O rders, D ivision, M ultiplication, A ddition и S ubtraction.

    Если вы используете BIDMAS, замените индикаторы O на индикаторы I .

    Вся суть BIDMAS и BODMAS в том, чтобы помочь вам запомнить, какие операции и в каком порядке выполнять.К сожалению, математика подсказывает, что это не порядок слева направо, как вы могли бы ожидать: давайте посмотрим на типичный пример и посмотрим, получите ли вы правильный ответ:

    2 + 3 х 4

    Вы пришли к выводу, что ответ — 20? Если это так, то вы выполнили расчет слева направо и сказали, что 2 плюс 3 равно 5 , и тогда умножить на 4, чтобы получить 20 . К сожалению, с математической точки зрения это неверно!

    Если вы посмотрите на правила BIDMAS / BODMAS, вы увидите, что умножение предшествует сложению (две операции, с которыми мы имеем дело), ​​и поэтому вы должны были иметь дело с умножением ДО , когда вы имели дело с сложением. Следовательно, правильная логика: 3, умноженное на 4, равняется 12 , а затем прибавляется к 2, чтобы получить 14 . 14 — правильный ответ.

    Я сбился со счета, сколько раз у меня был по телефону родитель, который говорил: «Вы просто должны ошибаться. Послушайте, я сделал это на своем калькуляторе и нажал 2 + 3 x 4, и он всегда, но всегда, дает мне ответ 20 ».

    Из многих тысяч вопросов на сайте образовательных викторин именно этот выпуск Порядка операций чаще всего вызывает запросы у наших учителей — и когда кажется, что у родителей есть калькуляторы, неудивительно, почему!

    Используются ли в экзамене 11-Plus BODMAS или BIDMAS?

    Вообще-то нет.Это может потребовать некоторой веры, но на экзамене 11-Plus неправильный ответ часто отмечается как правильный, в то время как правильный ответ отмечается вниз! Это связано с тем, что некоторые авторитеты считают, что дети в возрасте до 11 лет не должны знать Порядка операций, как это определено BIDMAS / BODMAS, и поэтому ожидают, что учащиеся будут ошибочно выполнять вычисления слева направо.

    Во избежание вероятности того, что кандидат 11-Plus будет оценен за правильный ответ (!), Крайне важно, чтобы вы выяснили в своей школе, следует ли применять правила Порядка операций в математических вопросах.

    Каковы правила BODMAS или BIDMAS?

    Чтобы увидеть, как правила BIDMAS (или BODMAS) применяются в более сложных сценариях, нам нужно проанализировать математическое выражение, которое содержит все различные элементы. Если вы готовы, то приступим!

    2 x 20 ÷ 2 + (3 + 4) x 3 2 — 6 + 15

    Шаг 1

    Разберитесь с скобами, чтобы получить:

    2 x 20 ÷ 2 + 7 x 3 2 — 6 + 15

    Шаг 2

    Сделайте с индексами (если вы думаете о BIDMAS) или ордерами (если вы думаете о BODMAS), чтобы получить:

    2 х 20 ÷ 2 + 7 х 9-6 + 15

    Шаг 3

    деление и умножение ранжируются одинаково, поэтому обработайте их вместе слева направо, чтобы получить:

    20 + 63 — 6 + 15

    Шаг 4

    Сложение и вычитание имеют одинаковый рейтинг, поэтому обработайте их вместе слева направо, чтобы получить:

    92

    Я надеюсь, что эта статья помогла вам разобраться в том, что такое BODMAS, BIDMAS и PEMDAS. Для более подробного ознакомления с порядком операций, а также с некоторыми интересными примерами, вы можете попробовать статью в Wiki по адресу: Порядок операций, но не ожидайте, что у вас не будет головной боли!

    Итак, BODMAS, BIDMAS и PEMDAS объяснили — есть ли что-нибудь еще, что вы хотели бы знать? Если у вас есть вопросы об образовании, просмотрите наш банк знаний. У нас есть множество статей, наполненных информацией, советами и советами для родителей. Это ценное оружие в арсенале любого родителя!

    Математика 1010 онлайн

    Отсутствующий оператор означает умножение

    По соглашению, когда оператор опускается, это означает умножение.Например, 3a означает 3 * a , а 3 (4 + 2) равно 18. xy означает x * y .

    (На самом деле есть исключение из этого правила в виде смешанные числа которые для нашей цели в основном бесполезны, и их следует избегать.)

    Последовательность операций

    По соглашению формулы вычисляются в следующей последовательности:

    1. Возведение в степень.
    2. Умножение и деление.
    3. Сложение и вычитание
    4. В случае операций того же уровня приоритета, оценка продолжается слева направо .

    Собственно, в приведенном выше списке должен быть пункт 0 перед всеми остальные: стандартные функции, такие как логарифмы, тригонометрические функции и т. д. Однако мы будем не используйте такие функции в Math 1010. (Но вы изучите их в большие детали в Math 1030, 1050 и 1060, а также в Calculus.)

    Если бы эти условности были абсолютными, нас бы сильно задушили, Чтобы предотвратить это бедствие, условные обозначения можно изменить, используя круглые скобки:

    • Сначала вычисляются выражения в скобках.
    Проиллюстрируем эти правила на нескольких примерах:
    • 2 + 3 * 4 = 2 + 12 = 14. Обратите внимание, что умножение проводится перед сложением.
    • (2 + 3) * 4 = 5 * 4 = 20. Вы используете круглые скобки для укажите, что добавление должно быть выполнено в первую очередь.
    • 10 — 4 — 2 = 6 — 2 = 4. Вы выполняете вычитания работают слева направо.
    • 10 — (4-2) = 10-2 = 8. Вы используете круглые скобки выполнить сначала крайнее правое вычитание, которое изменяет отвечать.2r

      Скобки могут быть вложенными , т. Е. пары совпадающих круглых скобок могут содержаться в других парах круглые скобки. Например,

      12- (6- (4-2)) = 12 — (6-2) = 12-4 = 8.

      Чтобы оценить формулы, содержащие вложенные пары круглых скобок, вы начните с самых сокровенных пар и продолжайте работать дальше.

      Поддержка правил PEMDAS Страница

      Добро пожаловать на страницу правил PEMDAS Math Salamanders.

      Здесь вы найдете помощь в использовании PEMDAS с некоторыми простыми инструкциями и множеством рабочих примеров.

      Mathway.

      PEMDAS обычно вводится примерно в 5-м классе и меняет то, как вы смотрите и читаете вычисления.

      Однако это не такое простое правило, как кажется — здесь есть несколько подводных камней, которые рассматриваются ниже!

      PEMDAS (иногда называемый PEMDAS, BEDMAS или BODMAS) является математическим сокращение, чтобы помочь вам понять, как работают вычисления.

      Аббревиатура относится к порядку, в котором должны выполняться любые вычисления, и обычно вводится примерно в пятом классе.

      Аббревиатура означает следующее:

      • P = скобки (скобки)
      • E = Показатели
      • M = умножение
      • D = Дивизион
      • A = Дополнение
      • S = вычитание

      Один из популярных способов запомнить правило PEMDAS — это поговорка:

      «Прошу прощения, моя дорогая тетя Салли».

      PEMDAS выглядит довольно просто из приведенного выше примера.

      Это говорит о том, что вам нужно сначала провести вычисления в круглых скобках (скобках), затем в экспонентах и ​​так далее …

      Однако правило PEMDAS не так однозначно, как кажется.

      Это связано с тем, что некоторые операции имеют одинаковый вес, например умножение и деление или сложение и вычитание.

      Вот почему некоторые люди вместо этого называют правило ПЕДМАС.

      Правило хорошо видно на диаграмме ниже.

      Это ясно показывает, что умножение и деление имеют равный вес. Это также показывает, что сложение и вычитание также имеют одинаковый уровень важности.

      PEMDAS за 4 простых шага

      Шаг 1) Сделайте круглые скобки (скобки).

      Шаг 2) Делаем любые экспоненты.

      Шаг 3) Произведите любое умножение ИЛИ деление слева направо

      Шаг 4) Выполните любое сложение или вычитание слева направо

      Итак, теперь мы знаем, что такое PEMDAS, пора посмотреть, как он работает…

      Вот

      Пример правила PEMDAS
      Пример 1

      3 + 4 х 5

      Это операции сложения и умножения.

      В PEMDAS умножение предшествует сложению.

      Итак, 3 + 4 x 5 = 3 + 20 = 23

      Пример 2

      4 х (2 + 3)

      Операции умножения и сложения, также есть круглые скобки (скобки).

      При использовании PEMDAS часть в круглых скобках должна стоять первой.

      Итак, 4 x (2 + 3) = 4 x 5 = 20

      Пример 3

      2 4 — (3 х 4)

      Есть экспоненты и круглые скобки, а также вычитание и умножение.

      При использовании PEMDAS часть в круглых скобках должна стоять первой.

      Итак 2 4 — (3 x 4) = 2 4 — 12

      При использовании PEMDAS, экспоненты идут дальше.

      Итак 2 4 — 12 = 16 — 12 = 4

      Пример 4

      12 ÷ 3 х 4

      Операции деления и умножения имеют одинаковый вес.

      Итак, мы просто выполняем операции слева направо.

      Итак, 12 ÷ 3 x 4 = 4 x 4 = 16

      Пример 5

      4–5 + 3

      Операции сложения и вычитания имеют одинаковый вес.

      Просто выполняйте операции слева направо.

      Итак, 4-5 + 3 = -1 + 3 = 2

      Пример 6 (сложнее)

      4 х (1 + 2) 2

      Есть экспоненты и круглые скобки, а также функции сложения и умножения.

      При использовании PEMDAS часть в круглых скобках должна стоять первой.

      Итак (1 + 2) = 3

      Далее идет экспоненциальная часть:

      3 2 = 9

      И, наконец, умножение:

      4 х 9 = 36

      Итак, 4 x (1 + 2) 2 = 4 x (3) 2 = 4 x 9 = 36

      Пример 7 (сложнее)

      6 x 4 ÷ 3 + 12 — 27 ÷ 3

      Здесь есть все четыре операции, поэтому нам нужно сначала выполнить умножение и деление.

      6 x 4 ÷ 3 мы можем делать слева направо, так как умножение и деление имеют одинаковый вес.

      6 x 4 ÷ 3 = 24 ÷ 3 = 8

      Это дает нам 8 + 12 — 27 ÷ 3

      Нам нужно сначала сделать деление.

      27 ÷ 3 = 9

      Остается 8 + 12 — 9.

      Мы можем выполнять это вычисление слева направо, так как все операции имеют одинаковый вес.

      8 + 12 — 9 = 20 — 9 = 11

      Таким образом, 6 x 4 ÷ 3 + 12-27 ÷ 3 = 24 ÷ 3 + 12-9 = 8 + 12-9 = 20-9 = 11

      Таблицы в этом разделе включают использование скобок и показателей в простых вычислениях.

      Существует также множество рабочих листов, предназначенных для практики и изучения PEMDAS.

      Использование этих листов поможет вашему ребенку:

      • знать и понимать, как работают круглые скобки;
      • понимает, как работают экспоненты в простых вычислениях.
      • понимает и использует PEMDAS для решения ряда проблем.

      Взгляните на еще несколько наших рабочих листов, похожих на эту тему.

      Листы в этой области помогут вашему ребенку понять использование и назначение знака равенства (=) в уравнении.

      Это также поможет детям научиться манипулировать и вычислять числовые выражения, чтобы они были эквивалентны.

      Это пригодится им, когда они начнут изучать алгебру и манипулировать алгебраическими уравнениями.

      Здесь вы найдете ряд распечатанных заданий по умственной математике для 5-го класса, которые понравятся вашему ребенку.

      Каждый рабочий лист проверяет детей по ряду математических тем, от числовых фактов до ментальной арифметики. к вопросам геометрии, дроби и меры.

      Отличный способ пересмотреть темы или использовать в качестве еженедельной викторины по математике!

      Саламандры по математике надеются, что вам понравятся эти бесплатные распечатываемые рабочие листы по математике. и все другие наши математические игры и ресурсы.

      Мы приветствуем любые комментарии о нашем сайте или рабочие листы в поле для комментариев Facebook внизу каждой страницы.


      Порядок операций

      Порядок действий — очень важный навык.Вы используете его каждый день, даже если не подозреваете об этом.

      Реальный пример PEMDAS

      Скажем, вы идете в супермаркет. Предположим, арахис стоит 3 доллара за фунт, а бутылка воды — 1 доллар. Вы получите 2 фунта арахиса и 1 бутылку воды.

      Сколько денег вы платите?

      Так как 1 фунт арахиса стоит 3 доллара, а вы купили 2 фунта, арахис стоит 6 долларов. Добавьте это к сумме, которую вы платите за воду (1 доллар), вы заплатили в общей сложности 7 долларов.

      Возможно, вы разобрались с этим без каких-либо серьезных проблем. Однако, если я представлю вам следующее уравнение, которое является моделью указанной выше проблемы, у вас может возникнуть тенденция прибавлять 3 к 1 и умножать результат на 2.

      2 × 3 + 1

      . 8, а оно не равно 7.

      Чтобы получить правильный ответ, вы должны сначала выполнить умножение.

      При выполнении вычислений всегда соблюдайте порядок операций и всегда выполняйте операции в соответствии со следующим правилом .

      Правило :

      1. Если используются символы группировки, например круглые скобки, сначала выполните операции внутри символов группировки.

      2. Вычислить любые выражения с показателем степени.

      3. Умножение и деление слева направо.

      4. Сложить и вычесть слева направо.

      Правило можно использовать, когда вы видите последовательно умножение, деление или и то, и другое, например, в следующей задаче:

      Пример № 1:

      4 2 — 6 × 2 ÷ 4 × 3 + 5

      Показатель Do:

      16-6 × 2 ÷ 4 × 3 + 5

      Умножение и деление слева направо

      16-12 ÷ 4 × 3 + 5

      16-3 × 3 + 5

      16-9 + 5

      Сложить и вычесть слева направо

      16-9 + 5

      7 + 5

      12

      Следующая мнемоника может помочь вам запомнить правило:

      PEMDAS (Прошу прощения, моя дорогая тетя Салли)

      P означает круглые скобки

      E означает экспоненты

      M означает умножить

      D. означает раздел

      . A означает сложение

      . S означает вычитание

      . Несмотря на то, что M стоит перед D в PEMDAS, две операции имеют одинаковую мощность. Точно так же, даже если A стоит перед S, две операции имеют одинаковую мощность.

      Еще примеры порядка работы


      Пример № 2:

      (2 + 5 2 ) + 4 × 3 — 10

      Сделать скобку:

      (2 + 25) + 4 × 3 — 10

      27 + 4 × 3 — 10

      Произвести умножение

      27 + 12-10

      Добавить

      39-10

      Вычесть

      29

      Пример № 3:

      10-14 ÷ 2 = 10-7 = 3 (Деление предшествует вычитанию )

      Помните, что если вы видите умножение и деление одновременно, выполняйте операцию слева направо.

      4 + 5 ÷ 5 × 6 = 4 + 1 × 6 = 4 + 6 = 10

      Порядок проведения викторины. Посмотрите, как вы понимаете этот урок.

      Рабочие листы для операций

      Добро пожаловать на страницу рабочих таблиц порядка операций на Math-Drills.com, где мы обязательно выполняем заказы! Эта страница включает рабочие листы Порядок операций с использованием целых, десятичных и дробных чисел.

      Учащиеся начальной и средней школы обычно используют аббревиатуры PEMDAS или BEDMAS, чтобы помочь им запомнить порядок, в котором они заполняют многооперационные вопросы. Буквы «P» или «B» в аббревиатуре обозначают круглые или квадратные скобки. Все операции в скобках выполняются первыми. «E» относится к любым показателям; все показатели рассчитываются после скобок. Буквы «M» и «D» взаимозаменяемы, так как умножение и деление завершаются в том порядке, в котором они появляются слева направо. Четвертый и последний шаг — найти сложение и вычитание в том порядке, в котором они появляются слева направо.

      В последнее время студентов учат аббревиатуре PEMA для обозначения порядка операций, чтобы избежать путаницы, присущей другим аббревиатурам.Например, в PEMDAS умножение предшествует делению, что, как некоторые ошибочно полагают, означает, что умножение должно выполняться перед делением в вопросе о порядке операций. Фактически, две операции выполняются в том порядке, в котором они выполняются в вопросе слева направо. Это признается в PEMA, который более правильно показывает, что в вопросе о порядке операций необходимо пройти четыре уровня.

      Если вы не хотите, чтобы ваши ученики занимались чем-то отличным от остального мира, было бы неплохо научить их понимать эти правила. Здесь не требуется никаких открытий или исследований. Это правила, которые необходимо выучить и применять на практике, и они были приняты в качестве стандартного подхода к решению любой многошаговой математической задачи.

      Самые популярные рабочие листы для порядка операций на этой неделе

      Порядок операций с листами с целыми числами

      Порядок операций с целыми числами на листах различной сложности.

      Порядок операций с целыми числами

      Ключи ответов для этих рабочих листов порядка операций показывают каждый шаг, поэтому легко определить, где учащиеся могли ошибиться, если они получат другой ответ.Ответы также можно использовать для моделирования того, как следует отвечать на вопросы, и для ознакомления студентов с порядком действий.

      Порядок операций с целыми числами
      (только сложение и умножение)

      Это хорошая отправная точка, когда используются только сложение и умножение (с добавлением нескольких круглых скобок). Эти рабочие листы помогут учащимся понять, что умножение выполняется до сложения, если не используются круглые скобки. Всегда приятно, если вы можете придумать несколько примеров, чтобы проиллюстрировать, что означают некоторые из этих вопросов. Например, 2 + 7 × 3 может относиться к количеству дней в двух днях и трех неделях. (9 + 2) × 15 может означать общую сумму заработка, если кто-то работал 9 часов вчера и 2 часа сегодня за 15 долларов в час.

      Порядок операций с целыми числами
      (только сложение, вычитание и умножение)
      Порядок операций с целыми числами
      (без показателей)

      Порядок операций с целыми листами

      Порядок операций с целыми листами с целыми числами как с отрицательными, так и с положительными целыми числами различной сложности.

      Порядок операций с целыми числами
      Порядок операций с целыми числами и
      без показателей

      Порядок действий с таблицами дробей

      Порядок операций с
      дробями

      Как и другие рабочие листы с порядком операций, рабочие листы с порядком дробей требуют некоторых предварительных знаний. Если ваши ученики борются с этими вопросами, вероятно, это больше связано с их способностью работать с дробями, чем с самими вопросами. Внимательно наблюдайте и постарайтесь точно указать, каких необходимых знаний не хватает, а затем потратьте некоторое время на изучение этих концепций / навыков, прежде чем продолжить. В противном случае приведенные ниже рабочие листы должны содержать довольно простые ответы и не должны приводить к чрезмерному выпадению волос.

      Порядок операций с десятичными знаками Рабочие листы

      Порядок операций с десятичными знаками на листах с положительными и отрицательными десятичными знаками и различной степенью сложности.

    Порядок выполнения математических действий

    Порядок выполнения математических действий

          В математике установлен определенный порядок выполнения математических действий при любой записи действий над числами. Для основных арифметических действий установлен следующий порядок: сначала выполняется возведение числа в степень, затем выполняется умножение и деление и в самую последнюю очередь выполняется сложение и вычитание.

           Если необходимо выполнить несколько действий умножения и деления, то выполняются они слева на право в том порядке, в котором записаны.

           Точно так же выполняются несколько действий сложения и вычитания: слева на право в том порядке, в котором действия сложения и вычитания записаны.

           Если хотят, чтобы порядок арифметических действий в какой-нибудь записи отличался от установленного, то употребляют скобки. Математические выражения заключают последовательно в круглые ( … ), квадратные [ … ( … ) … ] и фигурные { … [ … ( … ) … ] … } скобки. Действия над числами выполняются последовательно: сначала в круглых, затем в квадратных и, наконец, в фигурных скобках. Если в скобках заключены несколько различных математических действий, установленный порядок выполнения действий необходимо соблюдать: сначала выполняется умножение и деление, после этого сложение и вычитание внутри скобок. После получения результатов математических действий, заключенных в скобки, приступают к выполнению математических действий, записанных за скобками, соблюдая установленный порядок выполнения математических действий.

          Если деление обозначено чертой, необходимо сократить дробь, если это возможно. Деление, обозначенное чертой, выполняют после вычисления выражений, стоящих в числителе и в знаменателе.

          Знак извлечения корня рассматривается как запись при помощи скобок.

          При возведении в степень сначала выполняют все математические действия, указанные в показателе степени. Если требуется указать иной порядок действий, то употребляют скобки. В этом случае сперва выполняются все действия внутри скобок, только после этого приступают к выполнению действий за скобками.

          18 сентября 2009 года — 22 сентября 2019 года.

    © 2006 — 2021 Николай Хижняк. Все права защишены.

    Узнаем что сначала — сложение или умножение: правила, порядок выполнения действия и рекомендации

    С самого начала следует напомнить, чтобы потом не путаться: есть цифры – их 10. От 0 до 9. Есть числа, и они состоят их цифр. Чисел бесконечно много. Точно больше, чем звезд на небе.

    Математическое выражение − это записанное с помощью математических символов наставление, какие действия нужно произвести с числами, чтобы получить результат. Не «выйти» на искомый результат, как в статистике, а узнать, сколько их точно было. А вот чего и когда было − уже не входит в сферу интересов арифметики. При этом важно не ошибиться в последовательности действий, что сначала — сложение или умножение? Выражение в школе иногда называют «пример».

    Сложение и вычитание

    Какие же действия можно произвести с числами? Есть два базовых. Это сложение и вычитание. Все остальные действия построены на этих двух.

    Самое простое человеческое действие: взять две кучки камней и смешать их в одну. Это и есть сложение. Для того чтобы получить результат такого действия, можно даже не знать, что такое сложение. Достаточно просто взять кучку камней у Пети и кучку камней у Васи. Сложить все вместе, посчитать все заново. Новый результат последовательного счета камней из новой кучки − это и есть сумма.

    Точно так же можно не знать, что такое вычитание, просто взять и разделить кучу камней на две части или забрать из кучи какое-то количество камней. Вот и останется в куче то, что называется разностью. Забрать можно только то, что есть в куче. Кредит и прочие экономические термины в данной статье не рассматриваются.

    Чтобы не пересчитывать каждый раз камни, ведь бывает, что их много и они тяжелые, придумали математические действия: сложение и вычитание. И для этих действий придумали технику вычислений.

    Сумма двух любых цифр тупо заучиваются без всякой техники. 2 плюс 5 равно семь. Посчитать можно на счетных палочках, камнях, рыбьих головах – результат одинаковый. Положить сначала 2 палочки, потом 5, а потом посчитать все вместе. Другого способа нет.

    Те, кто поумнее, обычно это кассиры и студенты, заучивают больше, не только сумму двух цифр, но и суммы чисел. Но самое главное, они могут складывать числа в уме, используя разные методики. Это называется навыком устного счета.

    Для сложения чисел, состоящих из десятков, сотен, тысяч и еще больших разрядов, используют специальные техники − сложение столбиком или калькулятор. С калькулятором можно не уметь складывать даже цифры, да и читать дальше не нужно.

    Сложение столбиком −­­­­­­ это метод, который позволяет складывать большие (многоразрядные) числа, выучив только результаты сложения цифр. При сложении столбиком последовательно складываются соответствующие десятичные разряды двух чисел (то есть фактически две цифры), если результат сложения двух цифр превышает 10, то учитывается только последний разряд этой суммы – единицы числа, а к сумме следующих разрядов добавляется 1.

    Умножение

    Математики любят группировать похожие действия для упрощения расчетов. Так и операция умножения является группировкой одинаковых действий – сложения одинаковых чисел. Любое произведение N x M − есть N операций сложения чисел M. Это всего лишь форма записи сложения одинаковых слагаемых.

    Для вычисления произведения используется такой же метод – сначала тупо заучивается таблица умножения цифр друг на друга, а потом применяется метод поразрядного умножения, что называется «в столбик».

    Что сначала — умножение или сложение?

    Любое математическое выражение – это фактически запись учетчика «с полей» о результатах каких-либо действий. Допустим, сбора урожая помидоров:

    • 5 взрослых работников собрали по 500 помидоров каждый и выполнили норму.
    • 2 школьников не ходили на уроки математики и помогали взрослым: собрали по 50 помидоров, норму не выполнили, съели 30 помидоров, надкусили и испортили еще 60 помидоров, 70 помидоров было изъято из карманов помощников. Зачем брали с собой их в поле – непонятно.

    Все помидоры сдавали учетчику, он укладывал их по кучкам.

    Запишем результат «сбора» урожая в виде выражения:

    • 500 + 500 + 500 + 500 + 500 — это кучки взрослых работников;
    • 50 + 50 – это кучки малолетних работников;
    • 70 – изъято из карманов школьников (испорченное и надкусанное в зачет результата не идет).

    Получаем пример для школы, запись учетчика результатов работы:

    500 + 500 +500 +500 +500 + 50 +50 + 70 =?;

    Здесь можно применить группировку: 5 кучек по 500 помидоров − это можно записать через операцию умножения: 5 ∙ 500.

    Две кучки по 50 – это тоже можно записать через умножение.

    И одна кучка 70 помидоров.

    5 ∙ 500 + 2 ∙ 50 + 1 ∙ 70 =?

    И что делать в примере сначала − умножение или сложение? Так вот, складывать можно только помидоры. Нельзя сложить 500 помидоров и 2 кучки. Они не складываются. Поэтому сначала нужно всегда все записи привести к базовым операциям сложения, то есть в первую очередь вычислить все операции группировки-умножения. Совсем простыми словами — сначала выполняется умножение, а сложение уже потом. Если умножить 5 кучек по 500 помидоров каждая, то получится 2500 помидоров. А дальше их уже можно складывать с помидорами из других кучек.

    2500 + 100 + 70 = 2 670

    При изучении ребенком математики нужно донести до него, что это инструмент, используемый в повседневной жизни. Математические выражения являются, по сути (в самом простом варианте начальной школы), складскими записями о количестве товаров, денег (очень легко воспринимается школьниками), других предметов.

    Соответственно, любое произведение – это сумма содержимого некоторого количества одинаковых емкостей, ящиков, кучек, содержащих одинаковое количество предметов. И что сначала умножение, а сложение потом, то есть сначала начала вычислить общее количество предметов, а затем уже складывать их между собой.

    Деление

    Операция деления отдельно не рассматривается, она обратная умножению. Нужно что-то распределить по коробкам, так, чтобы во всех коробках было одинаковое заданное количество предметов. Самый прямой аналог в жизни – это фасовка.

    Скобки

    Большое значение в решении примеров имеют скобки. Скобки в арифметике – математический знак, используемый для регулирования последовательности вычислений в выражении (примере).

    Умножение и деление имеют приоритет выше, чем сложение и вычитание. А скобки имеют приоритет выше, чем умножение и деление.

    Все, что записано в скобках, вычисляется в первую очередь. Если скобки вложенные, то сначала вычисляется выражение во внутренних скобках. И это непреложное правило. Как только выражение в скобках вычислено, скобки пропадают, а на их месте возникает число. Варианты раскрытия скобок с неизвестными здесь не рассматриваются. Так делают до тех пор, пока все они не исчезнут из выражения.

    ((25-5) : 5 + 2) : 3 =?

    1. Это как коробочки с конфетами в большом мешке. Сначала нужно раскрыть все коробочки и ссыпать в большой мешок: (25 – 5 ) = 20. Пять конфет из коробочки сразу заслали отличнице Люде, которая приболела и в празднике не участвует. Остальные конфеты − в мешок!
    2. Потом связать конфеты в пучки по 5 штук: 20 : 5 = 4.
    3. Потом добавить в мешок еще 2 пучка конфет, чтобы можно было поделить на троих детей без драки. Признаки деления на 3 в данной статье не рассматриваются.

    (20 : 5 + 2) : 3 = (4 +2) : 3 = 6 : 3 = 2

    Итого: трем детям по два пучка конфет (по пучку в руку), по 5 конфет в пучке.

    Если вычислить первые скобки в выражении и переписать все заново, пример станет короче. Метод не быстрый, с большим расходом бумаги, зато удивительно эффективный. Заодно тренирует внимательность при переписывании. Пример приводится к виду, когда остается только один вопрос, сначала умножение или сложение без скобок. То есть к такому виду, когда скобок уже и нет. Но ответ на этот вопрос уже есть, и нет смысла обсуждать, что идет сначала — умножение или сложение.

    «Вишенка на торте»

    И напоследок. К математическому выражению не применимы правила русского языка – читать и выполнять слева направо:

    5 – 8 + 4 = 1;

    Это простенький пример может довести до истерики ребенка или испортить вечер его маме. Потому что именной ей придется объяснять второкласснику, что бывают отрицательные числа. Или рушить авторитет «МарьиВановны», которая сказала, что: «Нужно слева направо и по порядку».

    «Совсем вишня»

    В Сети гуляет пример, вызывающий затруднения у взрослых дяденек и тетенек. Он не совсем по рассматриваемой теме, что сначала — умножение или сложение. Он вроде как про то, что сначала выполняете действие в скобках.

    От перестановки слагаемых сумма не изменяется, от перестановки множителей тоже. Нужно просто записывать выражение так, чтобы не было потом мучительно стыдно.

    6 : 2 ∙ (1+2) = 6 ∙ ½ ∙ (1+2) = 6 ∙ ½ ∙ 3 = 3 ∙ 3 = 9

    Теперь точно все!

    Порядок выполнения действий / Справочник по математике для начальной школы

    1. Главная
    2. Справочники
    3. Справочник по математике для начальной школы
    4. Порядок выполнения действий

    В данном разделе мы познакомимся с порядком действий, с выражениями со скобками и без них.

     

    1) Если тебе нужно выполнить только сложение и вычитание или только умножение и деление, то все действия выполняют по порядку слева направо. 

    Например, 

    В числовом выражении 3 арифметических действия: сложение, вычитание и вычитание.

    Определим порядок действий и запишем их над арифметическими знаками: так как нет ни умножения ни деления, действия выполняют по порядку слева направо:

    Вычисляем:

    1) 10 + 15 = 25

    2) 25 — 6 = 19

    3) 19 — 8 = 11

    Полностью пример записываем так:

    10 + 15 — 6 — 8 = 25 — 6 — 8 = 19 — 8 = 11


    Например, 

    В числовом выражении 3 арифметических действия: деление, умножение и деление.

    Определим порядок действий и запишем их над арифметическими знаками: так как нет ни сложения ни вычитания, действия выполняют по порядку слева направо:

    Вычисляем:

    1) 15 : 5 = 3

    2) 3 • 4 = 12

    3) 12 : 6 = 2

    Полностью пример записываем так:

    15 : 5 • 4 : 6 = 3 • 4 : 6 = 12 : 6 = 2


    2) Если тебе нужно выполнить несколько арифметических действий (сложение, вычитание, умножение и деление), то сначала выполняют умножение и деление по порядку слева направо, а затем сложение и вычитание по порядку слева направо. 

    Например, 

    В числовом выражении 4 арифметических действия: вычитание, деление, сложение и умножение.

    Определим порядок действий и запишем их над арифметическими знаками: сначала производим деление, потом умножение, затем вычитание и сложение.

    1)15 : 3 = 5

    2) 6 • 8 = 48

    3) 10 — 5 = 5

    4) 5 + 48 = 53

    Полностью пример записываем так:

    10 — 15 : 3 + 6 • 8 = 10 — 5 + 6 • 8 = 10 — 5 + 48 = 5 + 48 = 53


    3) Если в выражении есть скобки, то сначала выполняют действия в скобках, но обязательно учитывать первое и второе правила.

    Например,

    В числовом выражении 4 арифметических действия: вычитание, деление, сложение и умножение.

    Определим порядок действий и запишем их над арифметическими знаками: сначала производим вычитание в скобках, затем деление, потом умножение и сложение.

    1) 25 — 10 = 15

    2) 15 : 3 = 5

    3) 6 • 8 = 48

    4) 5 + 48 = 53

    Полностью пример записываем так:

    (25 — 10) : 3 + 6 • 8 = 15 : 3 + 6 • 8 = 5 + 6 • 8 = 5 + 48 = 53


    Например

    В числовом выражении 4 арифметических действия: сложение, деление, сложение и деление.

    Определим порядок действий и запишем их над арифметическими знаками: сначала производим действия в скобках (деление, затем сложение), затем деление, потом сложение.

    1) 12 : 4 = 3

    2) 6 + 3 = 9

    3) 18 : 9 = 2

    4) 42 + 2 = 44

    Полностью пример записываем так:

    42 + 18 : (6 + 12 : 4) = 42 + 18 : (6 + 3) = 42 + 18 : 9 = 42 + 2 = 44

    Вывод: 

    Поделись с друзьями в социальных сетях:

    Советуем посмотреть:

    Скобки

    Правило встречается в следующих упражнениях:

    2 класс

    Страница 69. Вариант 2. Тест 1, Моро, Волкова, Проверочные работы

    Страница 111, Моро, Волкова, Степанова, Бантова, Бельтюкова, Учебник, часть 2

    Страница 40, Моро, Волкова, Рабочая тетрадь, часть 2

    Страница 68, Моро, Волкова, Рабочая тетрадь, часть 2

    Страница 12. Урок 4, Петерсон, Учебник, часть 2

    Страница 20. Урок 6, Петерсон, Учебник, часть 2

    Страница 88. Урок 37, Петерсон, Учебник, часть 2

    Страница 110. Урок 45, Петерсон, Учебник, часть 2

    Страница 3. Урок 1, Петерсон, Учебник, часть 3

    Страница 58. Урок 21, Петерсон, Учебник, часть 3

    3 класс

    Страница 35, Моро, Волкова, Степанова, Бантова, Бельтюкова, Учебник, часть 1

    Страница 93, Моро, Волкова, Степанова, Бантова, Бельтюкова, Учебник, часть 1

    Страница 31, Моро, Волкова, Рабочая тетрадь, часть 1

    Страница 60, Моро, Волкова, Рабочая тетрадь, часть 1

    Страница 69, Моро, Волкова, Рабочая тетрадь, часть 1

    Страница 21, Моро, Волкова, Степанова, Бантова, Бельтюкова, Учебник, часть 2

    Страница 43, Моро, Волкова, Степанова, Бантова, Бельтюкова, Учебник, часть 2

    Страница 54, Моро, Волкова, Степанова, Бантова, Бельтюкова, Учебник, часть 2

    Страница 59, Моро, Волкова, Степанова, Бантова, Бельтюкова, Учебник, часть 2

    Страница 34, Моро, Волкова, Рабочая тетрадь, часть 2

    4 класс

    Страница 94, Моро, Волкова, Степанова, Бантова, Бельтюкова, Учебник, часть 1

    Страница 5, Моро, Волкова, Рабочая тетрадь, часть 1

    Страница 45, Моро, Волкова, Рабочая тетрадь, часть 1

    Страница 66. Тест. Вариант 1, Моро, Волкова, Проверочные работы

    Страница 8, Моро, Волкова, Степанова, Бантова, Бельтюкова, Учебник, часть 2

    Страница 43, Моро, Волкова, Степанова, Бантова, Бельтюкова, Учебник, часть 2

    Страница 44, Моро, Волкова, Степанова, Бантова, Бельтюкова, Учебник, часть 2

    Страница 64, Моро, Волкова, Степанова, Бантова, Бельтюкова, Учебник, часть 2

    Страница 36, Моро, Волкова, Рабочая тетрадь, часть 2

    Страница 37, Моро, Волкова, Рабочая тетрадь, часть 2

    5 класс

    Задание 191, Виленкин, Жохов, Чесноков, Шварцбург, Учебник

    Задание 449, Виленкин, Жохов, Чесноков, Шварцбург, Учебник

    Номер 163, Мерзляк, Полонский, Якир, Учебник

    Номер 260, Мерзляк, Полонский, Якир, Учебник

    Номер 368, Мерзляк, Полонский, Якир, Учебник

    Номер 387, Мерзляк, Полонский, Якир, Учебник

    Номер 515, Мерзляк, Полонский, Якир, Учебник

    Номер 516, Мерзляк, Полонский, Якир, Учебник

    Номер 4, Мерзляк, Полонский, Якир, Учебник

    Номер 1124, Мерзляк, Полонский, Якир, Учебник

    6 класс

    Задание 73, Виленкин, Жохов, Чесноков, Шварцбург, Учебник

    Задание 92, Виленкин, Жохов, Чесноков, Шварцбург, Учебник

    Задание 373, Виленкин, Жохов, Чесноков, Шварцбург, Учебник

    Задание 378, Виленкин, Жохов, Чесноков, Шварцбург, Учебник

    Задание 400, Виленкин, Жохов, Чесноков, Шварцбург, Учебник

    Задание 411, Виленкин, Жохов, Чесноков, Шварцбург, Учебник

    Задание 417, Виленкин, Жохов, Чесноков, Шварцбург, Учебник

    Задание 422, Виленкин, Жохов, Чесноков, Шварцбург, Учебник

    Задание 445, Виленкин, Жохов, Чесноков, Шварцбург, Учебник

    Задание 454, Виленкин, Жохов, Чесноков, Шварцбург, Учебник


    © budu5. com, 2022

    Пользовательское соглашение

    Copyright

    Умножить или добавить сначала? Преподавание правил порядка действий

    Что на первом месте в порядке действий?

    Со временем математики согласовали набор правил, называемый порядком операций , чтобы определить, какую операцию выполнять первой. Когда выражение включает только четыре основные операции, действуют следующие правила:

    1. Умножать и делить слева направо.
    2. Сложение и вычитание слева направо.

    При упрощении выражения, такого как \(12 \div 4 + 5 \times 3 — 6\), сначала вычислите \(12 \div 4\), поскольку порядок операций требует сначала вычисления любого умножения и деления (в зависимости от того, что произойдет сначала) слева направо перед оценкой сложения или вычитания.В данном случае это означает, что сначала нужно вычислить \(12 \div 4\), а затем \(5 \times 3\). Как только все умножение и деление завершены, продолжайте складывать или вычитать (в зависимости от того, что наступит раньше) слева направо. Шаги показаны ниже.

    \(12 \дел 4 + 5 \умножить на 3 — 6\)
    \(3 + 5 \умножить на 3 — 6\) Потому что \(12 \дел 4 = 3\)
    \(3 + 15 — 6\) Потому что \(5 \х3 = 15\)
    \(18 — 6\) Потому что \(3 + 15 = 18\)
    \(12\) Потому что \(18 — 6 = 12\)

    Рассмотрим в качестве примера другое выражение:

    мы могли бы захотеть убедиться, что сложение или вычитание выполняется в первую очередь. Символы группировки , такие как круглых скобок \(( )\), скобок \([ ]\) или фигурных скобок \(\{ \}\), позволяют нам определить порядок, в котором выполняются определенные операции. выполненный.

    Порядок операций требует, чтобы операции внутри группирующих символов выполнялись до операций вне их.Например, предположим, что выражение 6 + 4 заключено в круглые скобки:

    .
    \(6 + 4 \times 7 — 3\)
    \(6 + 28 — 3\) Потому что \(4 \times 7 = 28\), что делается первым, потому что сначала оцениваются умножение и деление.
    \(34 — 3\) Потому что \(6 + 28 = 34\)
    \(31\) Потому что \(34 — 3 = 31\)
    \((6 + 4) \умножить на 7 — 3\)
    \(10 \умножить на 7 — 3\) Потому что \(6 + 4 = 10\), что и сделано во-первых, потому что это внутри круглых скобок.
    \(70 — 3\) Потому что \(10 \times 7 = 70\), и скобок больше нет.
    \(67\) Потому что \(70 — 3 = 67\)

    Обратите внимание, что выражение имеет совершенно другое значение! Что, если вместо этого мы заключим в скобки \(7 — 3\)?

    \(6 + 4 \умножить на (7 — 3)\)
    \(6 + 4 \умножить на 4\) На этот раз \(7 — 3\) в скобках, поэтому мы делаем это в первую очередь.
    \(6 + 16\) Поскольку \(4 \times 4 = 16\), и когда не осталось скобок, мы продолжаем умножение перед сложением.
    \(22\) Потому что \(6 + 16 = 22\)

    Этот набор скобок дает еще один ответ. Итак, когда задействованы круглые скобки, порядок операций следующий:

    1. Выполнять операции в скобках или группировать символы.
    2. Умножать и делить слева направо.
    3. Сложение и вычитание слева направо.

    Каков порядок операций? [Видео и практика]

    «Пожалуйста, извините, моя дорогая тетя Салли».

    Теперь я знаю, о чем вы думаете: «Что на самом деле означает эта фраза?» Совсем немного, потому что это высказывание дает ключ к запоминанию важной математической концепции: порядка операций.

    Порядок операций — одно из наиболее важных математических понятий, которое вы изучите, потому что оно диктует, как мы вычисляем задачи.Это дает нам шаблон, чтобы все решали математические задачи одинаково.

    Начнем с простого вопроса. Что такое операция?

    Операция — это математическое действие. Сложение, вычитание, умножение, деление и вычисление корня — все это примеры математической операции. Давайте посмотрим на эту задачу:

    \(7\times 4-6=?\)

    Выглядит просто, правда? Что ж, это было бы не так просто, если бы мы не понимали порядок, в котором происходит математическая операция.Если бы у нас не было правил, определяющих, какие расчеты мы должны произвести в первую очередь, мы бы пришли к другим ответам.

    Следует ли начать с вычитания 4 минус 6, а затем умножения на 7?

    Нет. Порядок операций говорит нам, как решить математическую задачу. И это возвращает нас к тете Салли.

    Каков порядок операций?

    Операции имеют определенный порядок, и это то, что «Пожалуйста, извините, моя дорогая тетя Салли» помогает нам понять. Это аббревиатура, которая говорит нам, в каком порядке мы должны решать математическую задачу.

    «Пожалуйста» означает « Скобки », поэтому сначала мы решаем все, что внутри скобок.

    Затем «Извините», что означает « Экспоненты ». Мы решаем это после того, как решим все в скобках.

    Умножение , то есть «Мое», и это происходит слева направо.

    А затем деление , то есть «Дорогой», что тоже бывает слева направо.

    А дальше у нас сложение и вычитание , что тоже бывает слева направо, а это «Тетя» и «Салли».

    Порядок операций Примеры

    Итак, теперь, когда мы знаем порядок операций, давайте применим его к нашей проблеме, которая у нас есть, и решим.

    \(7\times 4-6=?\)

    У нас нет круглых скобок и у нас нет показателей степени, но у нас есть умножение, поэтому мы делаем это до того, как будем выполнять сложение или вычитание. Давайте умножим \ (7 \ на 4 \). Это дает нам 28.

    \(28-6\)

    А теперь мы вычитаем 6, что дает нам 22.

    \(28-6=22\)

    Теперь давайте рассмотрим другую задачу.

    \(7+7\умножить на 3\)

    Без операций эту задачу можно вычислить как \(7+7=14\умножить на 3=42\).

    И это было бы неправильно!

    Помните, вы умножаете, прежде чем складывать. Следовательно, уравнение должно выглядеть так:

    \(7+(7\умножить на 3)\)

    \(=7+21\)

    \(=28\)

    Итак, когда мы решаем задачи таким образом, мы можем использовать круглые скобки, чтобы сгруппировать наши числа, которые будут занимать первое место.{2})\)

    \(=6\умножить на 9\)

    \(=54\)

    Видите? Решение уравнения в правильном порядке дает правильный ответ.

    Давайте попробуем еще одну задачу. Этот немного сложнее, но он прекрасно иллюстрирует порядок операций.

    \(5\умножить на 10-(8\умножить на 6\)\(-15)+4\умножить на 20\дел 4\)

    Запомнить порядок. Что мы делаем в первую очередь? Цифры в скобках. Итак, \ (8 \ times 6 = 48 \), затем мы вычитаем 15 и получаем 33.Вот как теперь выглядит задача:

    \(5\times 10-33+4\times 20\div 4\)

    Итак, наш следующий шаг — умножение и деление, поэтому давайте решим все наши задачи на умножение и деление и тогда посмотрим, что у нас осталось.

    \(50-33+80\дел 4\)

    \(50-33+20\)

    Теперь закончим со сложением и вычитанием, вот что у нас есть:

    \(50- 33+20\)

    \(=50-13\)

    \(=37\)

    И наш ответ 37!

    Есть исключение.Если в уравнении есть только одно выражение, вам не нужно соблюдать порядок операций.

    Вот несколько примеров одиночных выражений.

    \(10+10\): Ну, других операций нет, так что просто сложите их и получите 20. То же самое с вычитанием, умножением и делением. Все это отдельные выражения.

    Итак, ребята, это наше видео о порядке действий. Я надеюсь, что это было полезно!

    Увидимся в следующий раз!

    Порядок работы — PEMDAS

    Операции

    «Операции» означают такие действия, как сложение, вычитание, умножение, деление, возведение в квадрат и т. д.Если это не число, вероятно, это операция.

    Но когда вы видите что-то вроде…

    7 + (6 × 5 2 + 3)

    … какую часть нужно вычислить в первую очередь?

    Начать слева и идти направо?
    Или идти справа налево?

    Предупреждение: Вычислите их в неправильном порядке, и вы можете получить неверный ответ!

    Итак, люди давно договорились следовать правилам при расчетах, а они таковы:

    Порядок действий

    Сначала делайте что-то в скобках

      4 × (5 + 3) = 4 × 8 =

    32

     
      4 × (5 + 3) = 20 + 3 =

    23

    (неверно)

    Экспоненты (степени, корни) перед умножением, делением, сложением или вычитанием

      5 × 2 2 = 5 × 4 =

    20

     
      5 × 2 2 = 10 2 =

    100

    (неверно)

    Умножьте или разделите перед сложением или вычитанием

      2 + 5 × 3 = 2 + 15 =

    17

     
      2 + 5 × 3 = 7 × 3 =

    21

    (неверно)

    В противном случае просто идите слева направо

      30 ÷ 5 × 3 = 6 × 3 =

    18

     
      30 ÷ 5 × 3 = 30 ÷ 15 =

    2

    (неверно)

    Как я все это помню.

    .. ? ПЕМДАС!

    Р

    P арендес первый

    Е

    E экспоненты (т.е. степени и квадратные корни и т. д.)

    МД

    M умножение и D ivision (слева направо)

    КАК

    Добавление A и удаление S (слева направо)

    Разделение и умножение имеют одинаковый ранг (и идут слева направо).

    Сложение и вычитание рангов одинаково (и идут слева направо)

    Итак, сделайте так:

    После того, как вы сделали «P» и «E», просто идите слева направо, выполняя любую «M» или «D», как вы их найдете.

    Затем идите слева направо, выполняя любые «А» или «S», как вы их найдете.



    Вы можете вспомнить, сказав « P аренда E извините M y D ухо A unt S союзник».

    Или … Пухлые эльфы могут потребовать перекусить
    Попкорн Пончики каждый понедельник Всегда воскресенье
    Пожалуйста, ешьте мамины вкусные яблочные штрудели
    Люди повсюду принимают решения о суммах

    Примечание: в Великобритании говорят BODMAS (скобки, порядки, деление, умножение, сложение, вычитание), а в Канаде говорят BEDMAS (скобки, экспоненты, деление, умножение, сложение, вычитание). Все это означает одно и то же! Неважно, как вы это помните, главное, чтобы вы поняли это правильно.

    Примеры

    Пример: Как вычислить

    3 + 6 × 2 ?

    M Умножение до A Дополнение:

    Сначала 6 × 2 = 12 , затем 3 + 12 = 15


    Пример: Как вычислить

    (3 + 6) × 2 ?

    P арендес первый:

    Сначала (3 + 6) = 9 , затем 9 × 2 = 18


    Пример: Как вычислить

    12/6 × 3/2 ?

    M умножение и D ivision имеют одинаковый ранг, так что просто идите слева направо:

    Сначала 12/6 = 2 , затем 2 × 3 = 6 , затем 6/2 = 3

    Практический пример:

    Пример: Сэм бросил мяч прямо вверх со скоростью 20 метров в секунду, какое расстояние он пролетел за 2 секунды?

    Сэм использует специальную формулу, учитывающую влияние гравитации:

    высота = скорость × время — (1/2) × 9. 8 × время 2

    Сэм вводит скорость 20 метров в секунду и время 2 секунды:

    высота = 20 × 2 − (1/2) × 9,8 × 2 2

    Теперь расчеты!

    Начните с: 20 × 2 − (1/2) × 9,8 × 2 2

    Первые скобки: 20 × 2 − 0,5 × 9,8 × 2 2

    Тогда показатели степени (2 2 = 4): 20 × 2 − 0,5 × 9,8 × 4

    Затем умножается: 40 − 19,6

    Вычесть и ГОТОВО! 20.4

    Мяч достигает 20,4 метра через 2 секунды

    Экспоненты экспонентов …

    Что насчет этого примера?

    4 3 2

    Экспоненты особенные: они идут сверху вниз (сначала сделайте экспоненту сверху). Итак, мы вычисляем так:

    Начните с:   4 3 2
    3 2 = 3×3:   4 9
    4 9 = 4×4×4×4×4×4×4×4×4:   262144

    Так 4 3 2 = 4 (3 2 ) , не (4 3 ) 2 2

     

     

    И, наконец, что насчет примера с самого начала?

    Начните с: 7 + (6 × 5 2 + 3)

    Скобки сначала, а затем Показатель степени :7 + (6 × 25 + 3)

    Затем Умножить :7 + (150 + 3)

    Затем Добавить :7 + (153)

    Скобки заполнены: 7 + 153

    Последняя операция Добавить : 160

     

    297, 1566, 1572, 298, 1568, 381, 1570, 3854, 3855, 383

    Порядок действий: PEMDAS

    Пурпурная математика

    Если вас просят упростить что-то вроде «4 + 2×3», естественно возникает вопрос «Как мне это сделать? Потому что есть два варианта!» Я мог бы сначала добавить:

    4 + 2×3 = (4 + 2)×3 = 6×3 = 18

    . ..или я мог бы сначала умножить:

    4 + 2×3 = 4 + (2×3) = 4 + 6 = 10

    Какой ответ правильный?

    MathHelp.com

    Кажется, что ответ зависит от того, как вы смотрите на проблему.Но у нас не может быть такой гибкости в математике; математика не будет работать, если вы не можете быть уверены в ответе или если одно и то же выражение можно вычислить так, чтобы получить два или более разных ответа.

    Чтобы устранить эту путаницу, у нас есть некоторые правила приоритета, установленные по крайней мере еще в 1500-х годах, называемые «порядком операций». «Операции» — это сложение, вычитание, умножение, деление, возведение в степень и группировка; «порядок» этих операций указывает, какие операции имеют приоритет (о которых заботятся) перед какими другими операциями.

    Распространенным способом запоминания порядка операций является сокращение (или, точнее, «аббревиатура») «PEMDAS», которое превращается в мнемоническую фразу «Пожалуйста, извините, моя дорогая тетя Салли». Эта фраза означает и помогает запомнить порядок «круглых скобок, показателей степени, умножения и деления, сложения и вычитания». В этом листинге указаны ранги операций: скобки опережают показатели степени, которые опережают умножение и деление (но умножение и деление имеют один и тот же ранг), а умножение и деление опережают сложение и вычитание (которые вместе находятся в нижнем ранге).Другими словами, приоритет равен:

    .
    1. Скобки (упростить внутри них)
    2. Экспоненты
    3. Умножение и деление (слева направо)
    4. Сложение и вычитание (слева направо)

    Когда у вас есть куча операций одного ранга, вы просто действуете слева направо. Например, 15 ÷ 3 × 4 — это не 15 ÷ (3 × 4) = 15 ÷ 12, а скорее (15 ÷ 3) × 4 = 5 × 4, потому что, идя слева направо, вы получаете деление подпишись сначала.

    Если вы в этом не уверены, проверьте это на своем калькуляторе, запрограммированном на иерархию порядка операций. Например, введя приведенное выше выражение в графический калькулятор, вы получите:

    Используя вышеприведенную иерархию, мы видим, что в вопросе «4 + 2×3» в начале этой статьи вариант 2 был правильным ответом, потому что нам нужно выполнить умножение, прежде чем мы сделаем сложение.


    (Примечание: носители британского английского часто вместо этого используют аббревиатуру «BODMAS», а не «PEMDAS». BODMAS означает «скобки, порядки, деление и умножение, а также сложение и вычитание». Поскольку «скобки» — это то же самое, что и круглые скобки. и «порядки» совпадают с показателями степени, эти две аббревиатуры означают одно и то же. Кроме того, вы можете видеть, что «М» и «Д» в англо-английской версии поменялись местами; это подтверждает, что умножение и деление совпадают тот же «ранг» или «уровень».Канадцы, говорящие на английском языке, разделили разницу, используя BEDMAS.)

    Порядок операций был установлен, чтобы предотвратить недопонимание, но PEMDAS может создать собственную путаницу; некоторые учащиеся иногда склонны применять иерархию, как будто все операции в задаче находятся на одном «уровне» (просто идут слева направо), но часто эти операции не «равны». Во многих случаях это помогает работать с проблемами изнутри наружу, а не слева направо, потому что часто некоторые части проблемы находятся «глубже», чем другие части.Лучший способ объяснить это — привести несколько примеров:

    Мне нужно упростить термин с показателем степени, прежде чем пытаться добавить 4:

    Мне нужно упростить в круглых скобках, прежде чем я смогу пройти через экспоненту. Только тогда я могу сделать добавление 4.

    4 + (2 + 1) 2 = 4 + (3) 2 = 4 + 9 = 13

    • Упростить 4 + [–1(–2 – 1)]
      2 .

    Я не должен пытаться делать эти вложенные скобки слева направо; этот метод просто слишком подвержен ошибкам. Вместо этого я постараюсь работать изнутри наружу. Сначала я буду упрощать внутри фигурных скобок, затем упрощать внутри квадратных скобок, и только потом позабочусь о возведении в квадрат. После того, как это будет сделано, я наконец смогу добавить 4:

    .

    4 + [–1(–2 – 1)] 2

    = 4 + [–1(–3)] 2

    = 4 + [3] 2

    = 4 + 9

    = 13


    Использование квадратных скобок («[» и «]» выше) вместо круглых не имеет особого значения. Скобки и фигурные скобки (символы «{» и «}») используются, когда есть вложенные скобки, чтобы помочь отслеживать, какие скобки с какими идут. Различные символы группировки используются только для удобства. Это похоже на то, что происходит в электронной таблице Excel, когда вы вводите формулу, используя круглые скобки: каждый набор круглых скобок имеет цветовую кодировку, поэтому вы можете различить пары:

    .
    • Упрощение 4(
      –2 / 3 + 4 / 3 ).

    Сначала упрощу в скобках:

    Итак, мой упрощенный ответ:

    8 / 3

    На следующей странице есть больше примеров работы….


    URL-адрес: https://www. Purplemath.com/modules/orderops.htm

    ПОРЯДОК ДЕЙСТВИЙ

    ПОРЯДОК ДЕЙСТВИЙ

    Как посчитать 2 + 3 х 7? Ответ 35 или ответ 23? Чтобы знать правильный ответ, нужно знать правильный порядок действий в отношении сложения, вычитания, умножения, деления и т. д.

    Правило 20:
    Умножение и деление должны быть завершены до сложение и вычитание.

    2 + 3 х 7 = 2 + 21 = 23 — правильный ответ на поставленный выше вопрос.

    Как вычислить (2 + 3) x (7 — 3)? ответ 32, 20 или ответ 14? Чтобы знать правильный ответ, нужно знать правильный порядок операций сложения, вычитания, умножения, деления и скобок.

    Правило 21:
    Выражения в скобках рассматриваются как одно число и должен быть рассчитан в первую очередь.

    (2 + 3) х (7 — 3) = 5 х 4 = 20 — правильный ответ на вопрос выше. проблема

    Как бы вы вычислили [3 + 7 — (2 + 3 x 6) +2 x 5 -7 +1]?

    Правило 22:
    Если скобки заключены в другие скобки, работайте изнутри наружу.

    В выражении выражение (2 + 3 x 6) является самой внутренней скобкой и должно быть вычислено первым. 2 + 3 х 6 = 2 + 18 = 20.

    Теперь выражение изменено на .Следующая скобка, которую нужно вычислить, это 7 — 20 + 2 x 5 = 7 – 20 + 10 = — 13 + 10 = — 3.

    Теперь выражение сокращается до [3 + {-3} — 7 + 1] = 0 — 7 + 1 = — 6.

    Как бы вы рассчитали.

    Правило 23:
    В скобках указано упростить выражение в круглых скобках, прежде чем продолжить. Подразделение символ играет ту же роль, что и скобка. Он предписывает вам рассматривать количество над числителем, как если бы оно было заключено в скобки, и рассматривать количество под числителем, как если бы оно были заключены в еще одну скобку. Когда вы закончите это задача, у вас есть то, что кажется двумя дробями, которые вы добавляете. Не так! возможно написано и умножение должно быть завершено перед сложением в каждом скобка..

    = Оба скобки были упрощены. Теперь выполните умножение на урожай . Последнее, что нужно делать это дополнение.

    Если вы хотите больше примеров и задач для работы, нажмите на соответствующее слово.

    Правило 20
    Правило 21
    Правило 22
    Правило 23
    Меню Назад к простым дробям [Идентификация] [Факторизация целых чисел] [Сокращение дробей] [Умножение] [Разделение] [Строительные фракции] [Добавление] [Вычитание] [Порядок работы] С.Домашняя страница OS MATHematics

    Вам нужна дополнительная помощь? Пожалуйста, разместите свой вопрос на нашем S.O.S. Математика CyberBoard.

    Автор: Нэнси Маркус
    Copyright 1999-2022 MathMedics, LLC. Все права защищены.
    Свяжитесь с нами
    Математика Медикс, ООО. — П.О. Box 12395 — Эль-Пасо, Техас 79913 — США
    пользователей онлайн за последний час

    Заказ математических операций, БОДМАС | SkillsYouNeed

    Для расчета, который имеет только одну математическую операцию с двумя числами, это простой случай сложения, вычитания, умножения или деления, чтобы найти ответ.

    А как быть, когда есть несколько номеров и разные операции? Может быть, вам нужно делить и умножать или складывать и делить. Что вы делаете тогда?

    К счастью, математика — дисциплина, основанная на логике. Как это часто бывает, есть несколько простых правил, которые помогут вам определить порядок выполнения вычислений. Они известны как «Порядок операций» .


    Правила упорядочивания в математике — BODMAS

    BODMAS — полезная аббревиатура, указывающая порядок решения математических задач. Важно, чтобы вы следовали правилам BODMAS, потому что без них ваши ответы могут быть неверными.

    Аббревиатура BODMAS означает:

    • B ракетки (части расчета в скобках всегда идут первыми).
    • O заказы (числа, содержащие степени или квадратные корни).
    • D ivision.
    • M умножение.
    • Дополнение .
    • S вычитание.

    БОДМАС, БИДМАС или ПЕМДАС?


    Вы можете часто видеть BIDMAS вместо BODMAS. Они точно такие же. В BIDMAS «I» относится к индексам, которые аналогичны ордерам. Для получения дополнительной информации см. нашу страницу, посвященную специальным номерам и понятиям.


    ПЕМДАС

    PEMDAS обычно используется в США, он работает так же, как BODMAS. Акроним PEMDAS:

    P арендес,

    E экспоненты (силы и корни),

    M умножение и D ivision,

    Добавление и удаление S .



    Дополнительное чтение из навыков, которые вам нужны


    Руководство по необходимым навыкам счета

    Это руководство, состоящее из четырех частей, знакомит вас с основами счета от арифметики до алгебры, с промежуточными остановками на дробях, десятичных дробях, геометрии и статистике.

    Если вы хотите освежить свои знания или помочь своим детям в обучении, эта книга для вас.


    Использование БОДМАС

    Кронштейны

    Начните с чего угодно внутри квадратных скобок , двигаясь слева направо.

    Пример:

    4 × (3 + 2) = ?

    Нужно сделать операцию, в скобках сначала 3+2, потом умножить ответ на 4.

    3 + 2 = 5.
    4 × 5 = 20

    Если вы проигнорируете скобки и произведете расчет слева направо 4 × 3 + 2, вы получите 14. Вы можете видеть, как скобки влияют на ответ.

    Заказов

    Затем сделайте что-нибудь, связанное со степенью или квадратным корнем (они также известны как порядка ), снова работая слева направо, если их больше одного.

    Пример:

    3 2 + 5 = ?

    Сначала вам нужно рассчитать мощность, прежде чем вы сможете добавить 5.

    3 2 = 3 × 3 = 9
    9 + 5 = 14

    Деление и умножение

    После того, как вы выполнили какие-либо части вычислений с использованием скобок или степеней, следующим шагом будет деление и умножение .

    Умножение и деление имеют одинаковый ранг, поэтому вы работаете с суммой слева направо, выполняя каждую операцию в том порядке, в котором она указана.

    Пример:

    6 ÷ 2 + 7 × 4 = ?

    Сначала вам нужно выполнить деление и умножение, но у вас есть по одному тому и другому.

    Начните слева и двигайтесь вправо, что означает, что вы начинаете с 6 ÷ 2 = 3. Затем выполните умножение, 7 × 4 = 28.

    Ваш расчет теперь равен 3 + 28.

    Завершите вычисление сложения, чтобы найти ответ, 31 .

    Смотрите наши страницы: Умножение и Деление для получения дополнительной информации.

    Сложение и вычитание

    Последним шагом является вычисление любого сложения или вычитания . Опять же, вычитание и сложение имеют одинаковый ранг, и вы просто работаете слева направо.

    Пример:

    4 + 6 — 7 + 3 = ?

    Вы начинаете слева и идете поперек.

    4 + 6 = 10
    10 − 7 = 3
    3 + 3 = 6
    Ответ: 6 .

    См. наши страницы: Сложение и Вычитание , чтобы узнать больше.

    Собираем все вместе

    Этот окончательный рабочий пример включает в себя все элементы BODMAS.

    Пример:

    4 + 8 2 × (30 ÷ 5) = ?

    Начните с вычисления в скобках.

    30 ÷ 5 = 6
    Это дает вам 4 + 8 2 × 6 = ?

    Затем посчитайте порядки — в данном случае квадрат 8.

    8 2 = 64
    Ваш расчет теперь равен 4 + 64 × 6

    Затем перейти к умножению 64 × 6 = 384

    Наконец выполнить сложение.4 + 384 = 388

    Ответ: 388 .



    Тестовые вопросы BODMAS

    Правила BODMAS легче всего понять с помощью некоторой практики и примеров.

    Попробуйте эти расчеты самостоятельно, а затем откройте окно (щелкните символ + слева), чтобы увидеть работу и ответы.

    3 + 20 × 3

    В этом расчете нет скобок или порядков.

    1. Умножение предшествует сложению, поэтому начните с 20 × 3 = 60.
    2. Расчет теперь выглядит как 3 + 60

    Таким образом, ответ будет 63 .

    25 − 5 ÷ (3 + 2)

    1. Начните со скобок. (3 + 2) = 5.
    2. Расчет теперь выглядит как 25 − 5 ÷ 5
    3. Деление стоит перед вычитанием.5 ÷ 5 = 1,
    4. Расчет теперь выглядит как 25 − 1

    Таким образом, ответ будет 24 .

    10 + 6 × (1 + 10)

    1. Начните со скобок. (1+10) = 11.
    2. Расчет теперь выглядит как 10 + 6 × 11
    3. Умножение предшествует сложению. 6 × 11 = 66,
    4. Расчет теперь выглядит как 10 + 66.

    Таким образом, ответ будет 76 .

    5 (3 + 2) + 5 2

    Когда нет такого знака, как в этом вычислении, оператор является умножением, таким же, как запись 5 × (3 + 2) + 5 2 .

    1. Сначала выполните вычисления внутри скобок: (3 + 2) = 5.
    2. Это дает вам 5 × 5 + 5 2 .
    3. Следующий шаг — заказы, в данном случае квадрат. 5 2 = 5 × 5 = 25.Теперь у вас есть 5 × 5 + 25.
    4. Деление и умножение предшествуют сложению и вычитанию, поэтому ваш следующий шаг 5 × 5 = 25. Теперь вычисление выглядит так: 25 + 25 = 50.

    Ответ: 50 .

    (105 + 206) — 550 ÷ 5 2 + 10

    В этом есть все! Но не паникуйте. BODMAS по-прежнему применяется, и все, что вам нужно сделать, это отменить расчет.

    1. Начните со скобок.(105 + 206) = 311.
    2. Расчет теперь выглядит как 311 – 550 ÷ 5 2 + 10
    3. Далее, приказы или силы. В данном случае это 5 2 = 25,
    4. .
    5. Расчет теперь выглядит как 311 – 550 ÷ 25 + 10
    6. Далее, деление и умножение. Умножения нет, а деление 550 ÷ 25 = 22.
    7. Теперь расчет выглядит так: 311 – 22 + 10.
    8. Хотя у вас еще осталось две операции, сложение и вычитание имеют одинаковый ранг, поэтому вы просто выполняете слева направо.311 – 22 = 289 и 289 + 10 = 299.

    Ответ: 299 .

    7 + 7 ÷ 7 + 7 × 7 – 7 = ?

    Подобные проблемы часто ходят по сайтам социальных сетей с надписями типа «90% людей понимают это неправильно». Просто следуйте правилам BODMAS, чтобы получить правильный ответ.

    1. Здесь нет скобок и порядков, поэтому начните с деления и умножения.
    2. 7 ÷ 7 = 1 и 7 × 7 = 49.
    3. Расчет теперь выглядит как 7 + 1 + 49 – 7
    4. Теперь выполните сложение и вычитание. 7 + 1 + 49 = 57 – 7 = 50

    Таким образом, ответ будет 50 .


    Как дела?

    Надеюсь, вам удалось правильно ответить на все вопросы. Если нет, вернитесь и просмотрите, где вы ошиблись, и перечитайте правила еще раз.

    Чем больше вы практикуетесь, тем проще становится БОДМАС, и в конечном итоге вам даже не придется об этом думать.

    Порядок действий

    Если у вас есть математическая задача, которая включает более одной операции, например, сложение и вычитание или вычитание и умножение , что вы делаете в первую очередь?

    Пример #1 : 6 ? 3 х 2 = ?

    • Вы сначала выполняете вычитание (6 х 3 = 3), а затем умножение (3 х 2 = 6 )?
    • Или вы начинаете с умножения (3 x 2 = 6), а затем вычитаете (6 ? 6 = 0 )?

    PEMDAS

    В подобных случаях мы следуем порядку действий . Порядок, в котором должны быть сделаны операции, сокращаются как PEMDAS :

        1. E XPonents
        2. м Ultiplication и D Ivision (слева направо)
        3. DDITITION и S UBTRACTION (слева направо)

        (один из способов запоминания этого — подумать о фразе P аренду E XCUSE M Y D ухо A и S союзник.)

        • В приведенном выше примере мы имеем дело с умножением и вычитанием. M умножение происходит на шаг раньше S вычитания, поэтому сначала мы умножаем 3 x 2, а затем вычитаем сумму из 6, в результате чего получается 0.

        Пример #2 : 30 5 x 2 + 1 = ?

        • Нет P арентез.
        • Нет компонентов E .
        • Начнем с умножения M и ivision D , работая слева направо.
          ПРИМЕЧАНИЕ: Несмотря на то, что умножение предшествует делению в PEMDAS, они выполняются на одном шаге слева направо. Сложение и вычитание также выполняются на одном шаге.
        • 30 5 = 6 , остается 6 x 2 + 1 = ?
        • 6 х 2 = 12 , остается 12 + 1 = ?
        • Затем мы выполняем сложение A : 12 + 1 = 13

        Обратите внимание, что если бы мы выполнили умножение перед делением, то получили бы неправильный ответ:

        • 9 5 x 2 = 10

          , остается 30 10 + 1 = ?
        • 30 10 = 3 , остается 3 + 1 = ?
        • 3 + 1 = 4 (отклонение на 9!)

        Последний пример для продвинутых учащихся, использующий все шесть операций:

        2 х 3 6 ? 1 = ?

        • Начните с P арентез: 4 ? 2 = 2 .(Несмотря на то, что вычитание обычно выполняется на последнем шаге, поскольку оно заключено в скобки, мы делаем это первым.) Остается 5 + 2 2 x 3 6 ? 1 = ?
        • Затем E множителей: 2 2 = 4 .

    Добавить комментарий

    Ваш адрес email не будет опубликован. Обязательные поля помечены *

    2019 © Все права защищены. Карта сайта